

Hello,
Dr. Batman
Hello Doctor, Welcome!
Profile

Name: Batman
Email: batman@gotham.com
OBSTETRICS AND GYNAECOLOGY REVIEW
(Total Questions - 400)Q.1. A female patient with well-controlled DM wants to get pregnant. She asks you about the risk of congenital abnormalities. When would you advice her to start DM medications?
Correct Answer : A
Optimal pregnancy timing to start medication for diabetes should ideally be started or adjusted before pregnancy to ensure proper blood sugar control. High blood sugar levels in early pregnancy increase the risk of congenital abnormalities in the fetus as organogenesis occurs during the first 8 weeks.
Incorrect Options-
- First trimester- starting medication at this point might help but does not eliminate risks of abnormalities since the development of the fetus has already begun.
- Second/Third trimester- waiting until later in pregnancy delays treatment and causes complications for both mother and baby.
Congenital anamolies- neural tube defects [anencephaly, spina bifida], cardiac defects [VSD, TGA], caudal regression syndrome, renal malformation [agenesis urinary tract defects], gastrointestinal defects [small left colon syndrome].
Q.2. A pregnant lady wants to do screening tests but insists that she doesn't want any invasive procedure, what will you do?
Correct Answer : A
For a pregnant women who prefers non-invasive screening the best choice is ultrasound [USG]. Its completely safe as it uses sound waves,and provides valuable information without any risk to the fetus and mother.
Incorrect Options-
- Amniocetesis is an invasive procedure.
- Transvaginal ultrasound is not as risky but still involves inserting a probe and is considered invasive.
- Urine tests are not invasive but arent comprehensive enough to assess the condition of fetus.
Q.3. What is the risk of GDM on the mother's health later in life?
Correct Answer : B
Gestational diabetes [GDM] increases a mothers long term risk of developing type 2 DM. The hormonal change that causes insulin resistance durinng pregnancy may persist, leading to type 2 DM.
Incorrect Options-
- Type 1 DM - its an auto immune condition.
- Impaired fasting glucose- GDM may increase imapaired fasting glucose but the main concern here in progression to type 2 DM.
- No risk is an incorrect option since GDM is a known predictor for future health issues.
Q.4. What does clomiphene citrate do?
Correct Answer : A
Clomiphene citrate is used to induce ovulation. It tricks the body into thinking that the estrogen levels are low, prompting the release of hormones like FSH to stimulate egg maturation and induce ovulation.
Q.5. A pregnant lady with cardiac disease presents in labor, which one of the following would you NOT do?
Correct Answer : B
In a pregnant woman with cardiac condition, vaginal delivery is avoided as the strain of labour can put significant stress on the heart incresasing the risk of complications.
Other options-
- Epipidural anesthesia -helps manage pain and reduce stress, making it safer for women with a cardiac condition.
- Diuretics- can control fluid overload, reducing hearts workload.
- Digitalis- helps improve heart function when needed.
Q.6. What is the most common presentation of placenta previa?
Correct Answer : A
Placenta previa occurs when the placenta attaches low in the uterus, near or covering the cervix. The most common symptom of placenta previa is painless vaginal bleeding, often bright red in colour.
The following are features of placenta previa-
- Painless vaginal bleeding- sudden, bright red bleeding in the secind/third trimester.
- Soft uterus- no tenderness or contraction.
- Fetal malpresentation- breech or transverse lie due to placenta in the lower segment.
- High presenting part- fetal head remains unengaged.
- USG- placenta covers or is near the cervix.
Q.7. A pregnant lady, at 8weeks gestation presents with a history of bleeding for the last 12 hours with lower abdominal pain & h/o passing tissue. On examination the internal os was 1 cm dilated. What is the diagnosis?
Correct Answer : B
Vaginal bleeding, lower abdominal pain, passage of tissue, and a 1 cm dilated cervical os indicates that not all product of conception has been expellled.
Incorrect Options-
- Complete abortion- all products of conception expelled, bleeding subsides, os closed.
- Missed abortion- pregnancy is non viable but with no pain, bleeding, or tissue passage, and the os is closed.
- Molar pregnancy- excessive uterine enlargement with absent fetal parts.
Q.8. A young primigravida at 35 weeks gestation, has a BP of 140/90, headache, proteinuria & lower limb edema. What is the best management?
Correct Answer : D
The patient has signs of preeclampsia. Admission is necessary to monitor the mother and fetus, control BP, and prevent complication like eclamsia or HELLP syndrome.
Incorrect Options-
- Oral labetalol- used to control BP but is not sufficent enough and requires hospital monitoring.
- Diuretic- not recommended as edema in pregnancy is not due to fluid overload but vascular changes.
- Low sodum diet- dietary changes have minimal impact in managing acute preeclampsia.
Q.9. How should you treat an asymptomatic woman with trichomoniasis?
Correct Answer : C
Trichomoniaisis even in asymptomatic patient should be treated with antibiotics [eg., metronidazole or tinidazole] to prevent complications such as pelvic inflammatory disease, infertility, and transmission to sexual partners.
Q.10. A pregnant woman, multigravida, 28 weeks gestation is diagnosed with mild gestational diabetes after an abnormal glucose tolerance test. There is no history of glycosuria or macrosomia. What is the next step in management?
Correct Answer : C
In mild gestational diabetes during earlier gestation, dietary changes and lifestyle modifications are the first-line management to control blood sugar before considering medications or insulin.
Incorrect Options-
- Repeat glucose trolerance test- unnecessary once GDM is confirmed.
- C- section- not immmediately indicated unless there is poor glycemic control leading to complications like macrosomia.
- Sliding insulin helps manage sugar levels effectively in gestational diabetes that persistes late in pregnancy.
Q.11. A pregnant lady in her 30 weeks gestation diagnosed to have swine flu. She has high-grade fever and cough for 4 days, her RR= 25/min. What will you do for her?
Correct Answer : A
Swine flu [H1N1 influenza] in pregnancy can lead to severe complications, so antiviral treatment with oseltamivir [Tamiflu], should be started immediately without waiting for the test results.
Q.12. A 27-year-old pregnant lady, 19 weeks gestation, who is a smoker, presented with PV bleeding followed by painless delivery. She was told nothing was wrong with her or the baby. What is the diagnosis?
Correct Answer : A
The patient's presentation of painless vaginal bleeding at 19 weeks of gestation, followed by spontaneous delivery, strongly suggests cervical incompetence. This condition involves the premature dilation of the cervix in the absence of contractions and can lead to pregnancy loss in the second trimester. In cases of cervical incompetence, factors such as structural abnormalities or previous cervical trauma may contribute to diminished support for the pregnancy as it advances. Healthcare providers must comprehensively assess the patient's medical history and perform a thorough examination.
Additionally, considering potential interventions, such as cervical cerclage in subsequent pregnancies, may be beneficial in mitigating the risk of recurrence.
Q.13. A 30 year old woman in her third trimester of pregnancy developed a sudden massive swelling of her left lower extremity extending from the inguinal region to the ankle. What is the most appropriate sequence of work up & treatment?
Correct Answer : B
The sudden and significant swelling of the left lower extremity in a pregnant woman at 30 weeks of gestation is indeed highly suggestive of deep vein thrombosis (DVT).
Impedance plethysmography is a non-invasive test that assesses changes in blood volume in the legs, which can help in detecting DVT. While compression ultrasonography is the preferred initial diagnostic tool in pregnancy, impedance plethysmography can be considered when ultrasound is unavailable.
Low molecular weight heparin (LMWH) is the preferred blood thinner for treating deep vein thrombosis (DVT) in pregnant women. It is safe and effective in preventing the formation of new blood clots. LMWH has fewer side effects compared to other types of heparin and is easier to manage since it provides a more predictable response.
Bed rest is initially recommended for patients diagnosed with deep vein thrombosis (DVT) to help reduce the risk of embolization. By minimizing movement, bed rest can help prevent dislodging a blood clot, potentially leading to serious complications such as a pulmonary embolism. However, the management of DVT should be closely monitored by healthcare professionals to determine the appropriate course of treatment, which may include anticoagulation therapy alongside bed rest.
Q.14. A young female patient who is an office worker presented with vaginal itching associated with a greenish-yellowish vaginal discharge. Examination revealed red spots on the cervix. What is the diagnosis?
Correct Answer : A
Greenish vaginal discharge, the appearance of red spots = Trichomoniasis.
Incorrect options-
- Candidiasis- thick white, odorless discharge, and intense itching.
- Gonorrhea- can cause greenish-yellowish discharge but is not presented as red spots.
- Gardnerella vaginalis- thin grayish-white discharge with a fishy odor.
Q.15. A female patient presented with oligomenorrhea, having only 3 periods in the past year. She also had acne & hirsutism. Her body weight was 60 kg. PV examination was normal. What is the diagnosis?
Correct Answer : A
PCOS is a common hormonal imbalance leading to symptoms like irregular periods, acne, and hirsutism
The absence of other symptoms and normal pelvic examination further support the diagnosis.
Q.16. A 50-year-old post-menopausal woman who is taking estrogen OCP every month & stops at the 21st day of the cycle presents with vaginal bleeding in the form of spotting 2-3 days after stopping the estrogen OCP. What is the best management?
Correct Answer : B
Postmenopausal bleeding even when taking estrogen must be evaluated thoroughly to rule out serious causes like endometrial hyperplasia or carcinoma.
Incorrect option-
- Pap smear- it does not evaluate endometrium.
- Stop estrogen- discontinuing estrogen without determining the cause of bleeding would delay diagnosis and management.
- Continuing estrogen may lead to the masking of serious underlying conditions.
Q.17. Which of the following statements is correct about uterovaginal prolapse?
Correct Answer : A
Uterovaginal prolapse is a condition where the uterus or vaginal walls sag into the vaginal canal, leading to a sensation of pelvic pressure or heaviness. This feeling can become more pronounced during prolonged standing or physical activity due to the effects of gravity, which exacerbates the descent of pelvic organs. Women may also experience discomfort, urinary issues, or difficulty with bowel movements.
It's important for those experiencing these symptoms to consult a healthcare provider for an accurate diagnosis and potential treatment options.
Q.18. A couple is trying to have baby for the last 6 months of unprotected intercourse. They wanted to know the possible cause of their infertility. What will you do?
Correct Answer : A
Infertility is typically defined as the inability to conceive after 12 months of unprotected intercourse for women under 35 years old. For couples who have been trying for only 6 months, reassurance and encouragement to continue attempting conception are usually advised, unless there are specific risk factors present. These risk factors may include irregular menstrual cycles, known reproductive issues, or relevant medical histories that could affect fertility.
Q.19. A 34-year-old lady presents to the clinic with pelvic pain and menorrhagia. There is history of infertility, on examination the uterus was of normal size & retroverted; she had multiple small tender nodules palpable in the uterosacral ligament. What is the most likely diagnosis?
Correct Answer : B
The above-mentioned features align with endometriosis.
Incorrect option-
- Fibroids typically present with menorrhagia and may cause infertility but are often associated with an enlarged uterus.
- Adenomyosis usually presents with menorrhagia, an enlarged and tender uterus.
- PID presents with more acute symptoms like fever, purulent vaginal discharge cervical motion tenderness.
Q.20. Which of the following statement is true about an OCP?
Correct Answer : A
OCP primarily work by altering cervical mucus thickness,making it thicker and less penetrable to sperm, thus preventing fertilization.
Q.21. Which of the following options is the best indicator of labour progression?
Correct Answer : D
Dilation refers to the opening of the cervix, which is a direct indicator of the progress of labour. As the labour progresses the cervix dialates from 0- 10 cm
Descent refers to the downward movement of the fetus through the birth canal.
- while fetal heart rate and degree of pain are important for monitoring the well -being of the baby and the mother, they are not reliable indicators of labor progressions.
Q.22. Which of the following statement is true about OCP?
Correct Answer : A
OCP decreases the risk of ovarian cancer by inhibiting ovulation and limitingthe exposure of ovaries to hormones that could promote cancer development.
Incorrect Options-
- They do not increase the risk of breast cancer.
- OCPs are known to decrease the risk of endomentrial cancer because they reduce the number of menstrual cycles lowering the exposure of endometrium to estrogen.
- They do not increase the risk of ectopic pregnancy.
Q.23. What is the marker of ovarian cancer?
Correct Answer : A
CA- 125 is a protein that is often elevated with ovarian cancer.
Incorrect Options-
- HLA-DQ4 and HLA-B27 are associated with autoimmune disease.
- CA-19-9 is a marker primarily used for pancreatic cancer.
Q.24. What is the average length of the menstrual cycle?
Correct Answer : C
A tupical menstrual cycle ranges from 21-35 days, with 28 daysbeing considered the ''average'' cycle length.
Q.25. A pregnant ladies tests are negative antibodies for rubella and measles. What you will give her?
Correct Answer : D
Measles-mumps-rubella (MMR) vaccine and its components should not be administered to women known to be pregnant.
MMR VACCINE contains live attenuated viruses.
Q.26. What is true about antepartum hemorrhage?
Correct Answer : D
Antipartum hemorrhage refers to bleeding that occurs before labor, typically in the second or third trimester, and includes conditions like placental abruption and placenta previa.
Fetal mortality is generally higher than maternal mortality due to the risk of fetal hypoxia, premature delivery, or placental insufficiency.
Q.27. A pregnant lady came to antenatal clinic for routine checkup, her glucose tolerance test showed high glucose, diagnosed as gestational DM. What is the management?
Correct Answer : A
Nutritional advice- A well-balanced diet with controlled carbohydrate intake is crucial to managing blood glucose levels.
Incorrect Options-
- Insulin is given when blood glucose levels cannot be controlled with diet alone, oral hypoglycemic drugs are generally not recommended in pregnancy due to potential fetal risks, although drugs like metformin may be considered in some cases.
- Repeat OGTT is unecessary once GDM has been diagnosed.
Q.28. A pregnant lady with 7 cm dilated cervix, had induction of labor with oxytocin and artificial rupture of membrane. She is hypertensive and the baby is bradycardic. What you will do?
Correct Answer : A
Magnesium sulfate is the correct choice in the given scenario. The pregnant woman is hypertensive, and the baby is experiencing bradycatrdia which may indicate fetal distress. Magnesium sulfate is used in the management of severe hypertension to prevent eclampsia.
Incorrect Options-
- Give a dose of oxytocin- This is not appropriate in this case as the baby is already suffering from bradycardia and administering oxytocin would increase uterine contractions, which would further contribute in fetal distress.
- Give methotrexate- It is used for conditions like ectopic pregnancy not for labour management or fetal distress.
- Do nothing is not an acceptable approach.
Q.29. A 20-year-old pregnant female, exposed to rubella virus 3 days back, she was never vaccinated against rubella, mumps or measles. What would be the best thing to do for managing her?
Correct Answer : C
In most cases, the best course of action is to monitor the pregnancy closely and offer counselling about the potential risks of congenital rubella syndrome.
Incorrect options-
- IG is effective when given within 72 hours of exposure. However, in this case, the exposure occured 3 days ago, so it would be ineffective.
- Vaccine- [live attenuated] rubella vaccines are contraindicated during pregnancy due to potential risk of congenital rubella syndrome.
Q.30. What is the most common cause of postpartum hemorrhage?
Correct Answer : A
Uterine atony is the most common cause of postpartum hemorrhage. It occurs when the uterus fails to contract properly after childbirth, leading to excessive bleeding.
Incorrect options-
- Coagulation disorders can lead to postpartum hemorrhage but compared to uterine atony are less common.
- Retained placenta and rupture of uterus are serious complications of childbirth that can lead to significant bleeding, but are less common than uterine atony.
Q.31. A girl presented with amenorrhea for several months. Her BMI is 20 which is stable for past 5 years, what is the diagnosis?
Correct Answer : B
Pituitary adenoma is a benign tumor of the pituitary gland that can cause amenorrhoe due to disruption of hormone production.
- Pituitary adenoma→ increase prolactin→ decrease GnRH→decreae LH and FSH→decreased estogen→ Amenorrhea.
Incorrect options-
- Eating disorders can cause amennorhea, but the patient's stable BMI for the past 5 years makes this unlikely.
- Endometriosis and PCOS are common causes of amenorrhea, but they are often accompanied by other symptoms like- pelvic pain, hirsuitism, and irregular bleeding.
Q.32. Which of the following statements is true regarding GDM?
Correct Answer : A
Screening for GDM at 24-28 weeks is recommended practice for GDM screening during pregnancy. This is because GDM typically develop later in pregnancy due to increasing levels of placental hormones such as human placental lactogen, estrogen, and progesterone.
Incorrect options-
- Diet control is used as an initial treatment but, is always not successful. It might require addintional medications or insulin to manage the optimal levels .
- Screening for GDM at 8 or 12 weeks is not recommended because GDM typically develops later in pregnancy.
Q.33. A 48-years-old female with irregular menses presented with fatigue and no menstruation for 3 months, with increased pigmentation around the vaginal area. What is your next step in the management of this patient?
Correct Answer : C
An ultrasound is the most appropriate next step to assess the pelvic organs and rule out any underlying cause.
Q.34. A couple came for reversible contraception, the wife has previous h/o DVT. What method of contraception will you advise?
Correct Answer : B
IUD are safe and effective form of reversible contraception for a female with a history of DVT.
Incorrect option-
- Tubal ligation and vasectomy are permanent methods of contraception and does not align with the patients requirements.
- OCPs can increase the risk of blood clots , especially in women with a history of DVT.
Q.35. A female patient who underwent total vaginal hysterectomy with anterior & posterior repair now complains that urine is coming out through her vagina. What is the diagnosis?
Correct Answer : A
Uretrovaginal fistula is a communication between the ureter and vagina, which can occur as a complication of pelvic surgery, including hystertectomy. This condition allows urine to leak from the ureter into the vagina.
Q.36. A female who is currently on antibiotics complains of white cottage cheesy vaginal discharge. What is the diagnosis?
Correct Answer : A
Candida is a yeast infection that can cause a white, cottage cheese-like discharge. Antibiotics can disrupt the normal balance of bacteria in the vagina making it more suseptible to yeast overgrowth.
organism | color of discharge |
|
yellowish-green |
|
frothy green or yellow |
|
not associated with discharge |
Q.37. Which of the following statements is true regarding normal puerperium?
Correct Answer : D
Epidural analgesia can cause urinary retention due to numbing effect on the nerves that control the bladder.
Q.38. A lactating lady who hasn't taken the MMR vaccine wants to get vaccinated. What is the next step?
Correct Answer : C
She can take the vaccine without any further advice regarding breastfeeding, as MMR is a live attenuated vaccine and does not pose any risk to the breastfeeding infants.
Q.39. A female patient presents with signs of androgen excess and an ovarian mass, what is the likely tumor?
Correct Answer : A
Sertoli-Leydig cell tumors are rare tumors that produce androgens, leading to signs of androgen excess like hirsuitism, acne, and deepening of voice. They are often associated with ovarian masses.
Q.40. A girl presents with the complains of hirsutism, deep voice, and a receding hair line. What is the cause?
Correct Answer : A
The following symptoms described are suggestive of androgen excess. Androgens are male hormones that can cause masculinizing effects in the females.
Q.41. A pregnant lady at 16 weeks of gestation presents to the clinic, her ultrasound shows snowstorm appearance, what is the most likely diagnosis?
Correct Answer : A
Complete hydatidiform mole is a type of gestational trophoblastic disease GTD, where the placenta develops abnormally, forming grape like clusters. This abnormal development results in a snow storm appearance on ultrasound.
Q.42. A patient presents with whitish discharge from the nipple. Her investigations show pituitary adenoma, which hormone is responsible for this condition?
Correct Answer : A
Prolactin is a hormone produced by the pituitary gland. It is responsible for milk production in lactating women. Pituitary adenomas can cause production of prolactin in excess levels, leading to galactorrhea.
Q.43. A Young female presented with a previous history of fetal demise, now again she is in 34 weeks pregnant. What will you be doing as the next step in her management?
Correct Answer : B
In the abscence of any complications, it is generally safe to wait for spontaneous labor to start. Close monitoring of the mother and fetus is essential to ensure a safe delivery.
Q.44. Which of the following is the best medication for GDM?
Correct Answer : A
Insulin is the most common medication used for GDM. It is safe and effective treatment that helps control blood sugar levels during pregnancy.
Incorrect options-
- Metformin can be used but is not as effective as insulin.
- Sulfonylureas are not recommended as they can cause hypoglycemia in the new born.
- SGL-2 inhibitors are not currently recommended and are still under trial.
Q.45. Which of the following features are present in trichomoniasis?

Correct Answer : B
Greenish frothy discharge is a characterstick feature of trichomoniasis.
Q.46. A pregnant lady in her 3rd trimester came with bright red gush of blood, no abdominal pain or uterine tenderness. What is the diagnosis?
Correct Answer : A
Placenta previa is a condition where the placenta partially or completely covers the cervix. This can lead to painless vaginal bleeding, especially in the third trimester.
Incorrect options-
- Placenra abruption is a condition where placenta seprates prematurely from the uterine wall, causing bleeding and severe abdominal pain.
- Placenta accreta and increta are conditions where placenta is abnormally attached to the uterine wall.
Q.47. Which of the following is the cause of polyhydramnios?
Correct Answer : B
Duodenal atresia is a congenital condition in which the first part of the small intestine [duodenum] is blocked. This blockage prevents the fetus from swallowing and absorbing amniotic fluid.
Q.48. Which of the following is the most accurate diagnostic investigation for ectopic pregnancy?
Correct Answer : D
Laproscopy is considered the gold standard for diagnosing ectopic pregnancy.
Q.49. A 14-year-old female, presented with complains of history of lower abdominal pain since 6 months, pain is colicky in nature, radiates to the back and upper thigh, begins with onset of menses and last for 2-4 days, she missed several days of school during the last 2 months. Physical examination of abdomen and pelvis normal, normal secondary sexual character's development. What is the most likely diagnosis?
Correct Answer : A
This refers to painful periods that occur without an underlying medical condition. It typically starts during adolscence and improves with age.
Q.50. Nulligravida at 8 weeks of gestation, came for follow up and genetic screening, she refused for any invasive procedure but agrees for screening. What is the most appropriate action now?
Correct Answer : B
This combination is the most appropriate approach as it provides non- invasive screening for common chromosal abnormalities.
USG can assess fetal anatomy with gestational age, while maternal blood tests can screen for markers of down syndrome and other chromosomal abnormalities.
Q.51. Which of the following contraceptive is contraindicated during lactation?
Correct Answer : A
Increase estrogen→ decrease prolactin sceretion by inhibiting prolactin release from anterior pituitary→ decrease milk production →supressed lactation.
Q.52. A pregnant women presents with diphtheria. Which of the following is the correct course of action regarding vaccination?
Correct Answer : C
Diphtheria is a serious bacterial infection, vaccination with diphtheria toxoid is considered safe during pregnancy.
Q.53. Contraceptive pill that contains oestrogen increases risk for which of the following?
Correct Answer : A
There is a slight increase in the risk of breast cancer associated with combined oral contraceptive pills, particularly for women who have used them for a long time or at a young age.
Q.54. Which of the following is the best stimulus for breast milk secretion?
Correct Answer : B
The act of breastfeeding itself is the most effective stimulus for milk production. When a baby suckles, it transits signals to mothers brain, which triggers the release of hormones like oxytocin and prolactin. Oxytocin causes the milk to flow from the breasts, while prolactin stimulates milk production.
Q.55. A pregnant was diagnosed with UTI. Which of the following among the given option is the safest antibiotic?
Correct Answer : B
Ampicillin is generally considered safe for use during pregnancy.
Incorrect options-
- Ciprofloxacin- not recommended during pregnancy due to potential risks to the developing fetus including both bone and cartilage abnormalities.
- Tetracycline- can cause tooth discoloration,and developmental abnormalities in fetus.
- Cephalosporin- some drugs of this class have potential risks.
Q.56. A full term pregnant female with a wide pelvis, on delivery station +2, vertex position, CTG showed late deceleration. What is the most appropriate management?
Correct Answer : B
Late deceleration on CTG indicates fetal distress, which is suggestive of an immediate delivery to prevent further harm to the baby. Suction is a quick and effective method to assist in vaginal delivery in such situations.
Incorrect options-
- C-section- can be considered in some cases but is a more invasive procedure.
- Forceps delivery- is another option for assisted vaginal delivery, but it carries risks.
- Spontaneous delivery- unlikely to be sucessful in this situation.
Q.57. Endometriosis is best diagnosed by which of the following diagnostic method?
Correct Answer : B
Laproscopy is considered a gold standard for diagnosing endometriosis. It involves inserting a thin, lighted tube with a camera into abdomen through a small incision. This allows direct visuslization of the pelvic organs and identification of endometrial tissue which is located outside the uterus.
Q.58. A 41 weeks pregnant women's biophysical profile showed oligohydramnios. She has no complaints except mild HTN. What is the appropriate management?
Correct Answer : C
Oligohydramnios at 41 weeks is a concern as it is indicative of placental insufficiency and compromise of fetal well-being.Therefore inducing labor is the correct management to ensure the wellbeing of both mother and baby, which would otherwise be compromised if other options are used for further management.
Q.59. A young female presents with whitish-grey vaginal discharge, a positive KOH test, and has a fishy odor. What is the diagnosis?
Correct Answer : B
Bacterial vaginosis has a combination of classic features which includes whitish-grey discharge, a positive KOH test [which release a fishy odor when mixed with vaginal discharge], and a fishy odor itself. This condition is caused by an imbalance of bacterial flora in the vagina.
Q.60. A female patient complains of odorless and colorless vaginal discharge that appears after intercourse. What is the treatment?
Correct Answer : B
This is the most appropriate treatment for a post-coital odorless and colorless discharge. Douching after intercourse can help remove any irritants or bacteria that may have been introduced during intercourse.
Q.61. Which of the following options regarding primary dysmenorrhea is true?
Correct Answer : C
NSAIDs are effective in relieving pain associated with primary dysmenorrhea. They work by reducing the production of prostaglandins, which are substances that cause uterine contractions and pain.
Q.62. Which of the following options is true regarding obstructed labor?
Correct Answer : D
Obstructed labor is a condition that arises during labor when the baby cannot pass through the birth canal due to various reasons like a small pelvis, large baby, or abnormal fetal position.
Incorrect options-
- Common in primi- While primiparous women may have a higher risk of obstructed labor,it can occur regardless of parity.
- Excessive caput and molding- it can sometimes be a sign of prolonged labor, they are not specific to obstructed labor.
- Most common occipto- anterior- this is the most common fetal position. However, this presentation is not necessary for preventing obstructed labor.
Q.63. What is the golden period to start breast feeding after delivery?
Correct Answer : A
The WHO [world health organization] has recommended initiating the breast feeding within the first hour of birth. This is known as the ''golden hour''. Early breastfeeding helps establish a strong bond between the mother and baby, promotes milk production, and provides essential nutrients to the newborn.
Q.64. A pregnant women has fibroid. Which of the following statements is true?
Correct Answer : B
Fibroids often shrink or even disappear after pregnancy and childbirth. The changing hormone levels during pregnancy and after delivery can contribute to the regression of its size.
Incorrect options-
- Presented with severe anemia- it can cause heavy mestrual bleeding but this is always not the case.
- Suregery is generally not recommended for fibroids during pregnancy unless they are causing any severe complicayion like heavy bleeding or obstruction.
- Presented with antepartum hemorrhage- they can increase the risk of APH but its not alwasy the case.
Q.65. A 28-year-old women presents with a history of amenorrhea since 7 weeks, she also complains of lower abdominal pain, she has a positive home pregnancy test, and comes with light bleeding. What is the next step in her management?
Correct Answer : B
The most appropriate next step is to confirm the pregnancy with a quantitative serum hCG test.
Q.66. A Female patient came with generalized abdominal pain. On examination you found suprapubic tenderness. On PV examination, there is tenderness on moving cervix and tender adnexa. What is the diagnosis?
Correct Answer : A
PID is an infection of the female reproductive organs, often caused by sexually transmitted bacteria. It typically presents with lower abdominal pain, tenderness on pelvic exam, and abnormal vaginal discharge.
The primary causative organism include-
- Chlamydia trachomatis- the most common bacterial cause of PID.
- Neisseria gonorrhoea
- Anaerobic bacteria- bacteroides species and peptostreptococcus
- Mycoplasma
Q.67. A 50-year-old female gives history of hot flushes. What is the best drug to reduce these symptoms?
Correct Answer : D
These medications are commonly used to treat hot flushes, especially when HRT is not suitable.Venlafaxine and paroxetine drugs belon to the class of antiderpressants that can also help in regulating body temprature, while clonidine is a medication used in controlling blood pressure that can also reduce hot flushes.
Q.68. Incase of forceful intercourse, the rupture of hymen occurs in which direction?
Correct Answer : A
The hymen is a thin membrane that partially covers the vaginal opening and is located at the 6 o'clock position. In case of a forceful sexual intercourse it is most likely to rupture in this direction.
[Hymen can rupture due to various reasons, including physical activies like sports or accidents].
Q.69. A pregnant female at 34 weeks presents with abdominal pain radiating to the back. O/E: fetus has transverse lie, back down & PV examination revealed open cervix 3 cm & plugging of bag. What's the management?
Correct Answer : A
In this scenario c-section would be the best management. The combination of transverse lie, back down presentation, and an open cervix with bulgingbag of waters indicates that vaginal delivery is unlikely to be sucessful.
Q.70. A patient presents with complains of yellowish vaginal discharge and itching. On swab and culture, the organism is identified as Trichomonas vaginalis. Which of the following is correct regarding further management of this patient?
Correct Answer : A
Metronidazole is the first-line treatment for trichomoniasis. It effectively kills the parasite and clears the infection.
Incorrect options-
- Clindamycin-it's an alternative treatment but not first-line.
- No need to treat husband- both sexual partners should be treated to prevent reinfection.
- Vaginal swab culture after 2 weeks- not necessary after treatment with metronidazole.
Q.71. A woman comes with right lower abdominal tenderness with no signs of infection and HCG her results are normal. What is the most likely diagnosis?
Correct Answer : A
Ovarian torsion is a condition in which the ovary twists over its supporting ligamens, cutting off blood supply.This can lead to sudden severe pain in the lower abdomen, often on one side.
Incorrect options-
- Endometriosis- causes choronic pelvic pain that worsens during menstruation.
- PID- presents with fever, vaginal discharge, and cervical motion tenderness.
- PCOS- It can cause irregular periods, acne, hirsuitism, but doesn't typically present with sudden, severe abdominal pain.
Q.72. A couple is asking for methods of emergency contraception, which one of the following would you suggest?
Correct Answer : A
Levonorgestrel is a progesterone only pill that is highly effective in preventing pregnancy within 72 hours of unprotected sex. It works by preventing ovulation, thickening cervical mucus, making it difficult for sperm to reach the egg.
Q.73. A Patient has history of amenorrhea for past 6 weeks. She now presented with abdominal pain. On examination, there is fluid in pouch of Douglas & spotting. What's the most likely diagnosis?
Correct Answer : A
Ruptered ectopic pregnancy is the most likely diagnosis in the given scenario. An ectopic pregnancy is defined as implantation of fertilized egg outside the uterine cavity, often in the fallopian tube. As the pregnancy progresses the tube can rupture, causing severe abdominal pain an internal bleeding.
Q.74. A patient has a history of cervical incompetence, she is now pregnant for 8 weeks. What's the management?

Correct Answer : A
This patient has a previous history of cervical incompetence and is therefore at a high risk of premature labor and delivery. Cerclage is a surgical procedure where a stitch is placed around the cervix to strengthen it and prevent premature opening. The suture is removed typically at 36-37 weeks. This helps to maintain the pregnancy and reduce the risk of premature birth.
Q.75. A lactating mother wants to get vaccinated with pox virus vaccine. Which of the following advice will you give her?
Correct Answer : C
Live vaccine like the pox virus vaccine should be avoided during breastfeeding as the vaccine virus can potentially be transmitted to the infant through breast milk. However if the mother already had an episode of pox virus infection,its generally safe to give the vaccine after 2-3 weeks of recovery, as the risk of transmission to the infant is minimal at this point.
Q.76. Which of the following statements regarding injectable progesterone is true?
Correct Answer : B
Injectable progesterone commonly used for contaception is known to cause irregular bleeding pattern and potential weight gain.
Q.77. A pregnant ladies thyroid function test shows high TBG & T4 and normal T3. What is this due to?
Correct Answer : A
During pregnancy there is an increase in thyroid hormone binding globulin [TBG] production by the liver. This leads to an increase in the T3 and T4 levels, as more T4 is bound to the TBG. However, the free T4 levels remain normal, as the increase in TBG is proportional to the increase in total T4.
Q.78. A pregnant lady gives history of increased body weight about 3 kgs from the last visit and lower limb oedema. How will you confirm she has pre-eclampsia?
Correct Answer : A
For confirming pre-eclampsia we need to monitor the BP, as it is characterized by high blood pressure and protein in the urine. Measuring the blood pressure is the most straight forward way to assess pre-eclampsia.
- Fetures of pre-eclampsia-
- HTN > or = to 140/90 mmhg measured on 2 occassions 4 hrs apart.
- Proteinuria- >/= 300mg/24 hr urine collection, protein-to-cretinine ration>/=0.3, urine dipstick>/=1+
- End organ dysfuntion with or without proteinuria
- intrauterine complications- IUGR, oligohydamnios.
Q.79. A pregnant women with 32 weeks pregnancy and no abnormalities visits you at the clinic. What should you advice the woman?
Correct Answer : A
Generally a normal pregnancy is expected to continue until 39-40 weeks. However, if there are no complications and if the mother desires, inducing labor at 36 weeks is generally considered safe, and it can reduce the risk of post term pregnancy complications.
Incorrect options-
- At 34/35 weeks- not generally recommended unless some specific medical reason like preeclampsia or fetal growth restriction occurs.
- At 37 weeks- it's a ressonable option in some cases but generally preferable to wait until 38-40 weeks.
Q.80. A Female patient 35-year-old with history of thromboembolic disease. What type of reversible contraceptive can she use?
Correct Answer : C
IUCD are a safe and effective long-term reversible contraceptive method. They are particularly suitable for women with a history of thromboembolic disease because they do not contain hormone, which can increase the risk of blood clots.
Q.81. A 36th weeks pregnant woman came with 7 cm cervical dilatation at zero station. During birth, CTG shows late deceleration. What is the management?

Correct Answer : B
Late decelaration on CTG indicates fetal distress, likely due to increased oxygen delivery to the fetus. The immediate management is to increase oxygenation to the fetus.
Q.82. A pregnant woman at 28 weeks of gestation was admitted to the hospital, during her stay the patient next to her develops chickenpox. She comes to you asking for advice as she's worried, you find that she is seronegative for varicella antibody. What will be your management?
Correct Answer : A
VZIG is a medication containing antibodies against the varicella zoster virus. It typically is used for immunocompromised individuals or pregnant women who have been exposed to chickenpox.
Q.83. A female patient came with the complain of severe vaginal bleeding. What is the most appropriate management?
Correct Answer : D
In a patient with severe vaginal bleeding a comprehensive approach is necessary to stabalize the patient and control the bleeding.
Q.84. Increase frequency of menses is called?
Correct Answer : B
- Polymenorrhea- increased frequency of mensturation.
- Metrorrhagia- bleeding between menstrual periods.
- Hypermenorrhea- heavy menstrual bleeding.
- Hypomenorrhea- scanty menstrual bleeding.
Q.85. A female presents with complains of itching, and cheesy white odorless discharge from vagina. What is the treatment?
Correct Answer : A
The symptoms described are characteristic of a vaginal yeast infection, commonly caused by Candida albicans, miconazole is an antifungal medication that is effective in treating vaginal yeast infection.
Q.86. A pregnant lady G1P0 at 13 week weeks of gestation presents to the clinic, she looks anxious, but is happy about her pregnancy, her blood pressure is 142/96, she does exercise 4-5 times / week, she denies any previous medical problem. What is the most likely diagnosis?
Correct Answer : C
Gestational hypertension (formerly known as pregnancy-induced hypertension): Idiopathic hypertension without significant proteinuria (< 300 mg/L) that develops at > 20 weeks’ gestation.
As many as 25% of patients may go on to develop preeclampsia.
Chronic hypertension: Present before conception and at < 20 weeks’ gestation, or may persist for >12 weeks postpartum.
Up to one-third of patients may develop superimposed preeclampsia.
- Preeclampsia: New-onset hypertension (SBP >/= 140 mmHg or DBP >/= 90 mmHg) and proteinuria (> 300 mg of protein in a 24-hour period) occurring at > 20 weeks’ gestation.
- Eclampsia: New-onset grand mal seizures in women with preeclampsia.
Q.87. A woman at 34 weeks of gestation presented with antepartum hemorrhage, she was conscious no other symptoms, what is the most likely cause?
Correct Answer : D
Since the patient is 34 weeks pregnant and has no other symptoms, placenta previa is the most likely diagnosis.
Q.88. Chronic use of oestrogen is associated with which of the following?
Correct Answer : A
Long term use of estrogen therapy has been linked to an increased risk of developing breast and endometrial cancer.
Q.89. A pregnant woman at 36 weeks, presented with agitation, BP: 88/60, and fetal distress. What is the diagnosis?
Correct Answer : B
Amniotic fluid embolism- Typically occurs acutely during labor and delivery or immediately at postpartum.
Classic signs and symptoms are hypoxia, hypotension with shock, altered mental status and disseminated intravascular coagulation. Other signs and symptoms include seizure activity, agitation, and evidence of fetal distress.
- Diagnosis: primarily a clinical diagnosis of exclusion.
- Treatment: Aggressive supportive management is needed, and the patient should be intubated and monitored.
Q.90. Which condition is not associated with increased alpha-feto protein?
Correct Answer : A
Breech presentation is not associated with increased alpha feto protein levels.
Q.91. A 34-year-old patient presented with amenorrhoea. G3 P3, all her deliveries were normal except after the second one she underwent D&C. Labs are all normal except: high FSH, high LH, low estrogen. What is the diagnosis?
Correct Answer : A
Ovarian faliure is a condition where the ovaries stop functioning properly before the age of 40. The elevated FSH and LH levels in this patient are indicative of ovarian faliure.
Q.92. A patient presents with h/o salpingitis presents with fluctuant swelling in posterior fornix. What is the management?
Correct Answer : A
Culdocentesis refers to the extraction of fluid from the rectouterine pouch posterior to the vagina through a needle > The Rectouterine Pouch is often reached through the posterior fornix of the vagina. The process of creating the hole is called "colpotomy" a scalpel incision is made to drain the fluid rather than using a needle. Drainage of a tubo-ovarian/pelvic abscess is appropriate if the mass persists after antibiotic treatment; the abscess is > 4–6 cm, or the mass is in the cul-de-sac in the midline and drainable through the vagina. If the abscess is dissecting the rectovaginal septum and is fixed to the vaginal membrane, colpotomy drainage is appropriate. If the patient’s condition deteriorates, perform an exploratory laparotomy.
Q.93. Case of Salpingitis and PID on penicillin but does not improve. What is the most likely organism?
Correct Answer : B
Outpatient antibiotic regimens : Regimen A: Ofloxacin or levofloxacin × 14 days +/– metronidazole × 14 days. Regimen B: Ceftriaxone IM × 1 dose or cefoxitin plus probenecid plus doxycycline × 14 days +/– metronidazole × 14 days. Inpatient antibiotic regimens: Cefoxitin or cefotetan plus doxycycline × 14 days. Clindamycin plus gentamicin × 14 days.
Q.94. A 29-year-old lady has a B-HCG 160. She complains of vomiting & abdominal pain. Which is more accurate to diagnose the cause?
Correct Answer : B
Pelvic ultrasound is the most accurate initial diagnostic tool.It helps in visualizing the gestational sac, confirm the location of pregnancy, and identify any potential complications.
Q.95. A female has breech presentation at 34 weeks, what will you do as further management?
Correct Answer : B
External cephalic version [ECV] ia a procedure to turn a breech baby into a head down position. It is typically performed between 36-37 weeks.
Q.96. A 28-year-old lady G3+P3 complains of infertility and a H/o diffuse abdominal pain. What is the cause?
Correct Answer : A
Uterine fibroids are noncancerous growths in the uterus. They can cause a variety of symptoms, including infertility, heavy menstrual bleeding, pelvic pain, and pressure on bladder and rectum. The diffuse abdominal pain mentioned in the question is a common symptom of uterine fibroid.
Q.97. A female lady after delivery developed pelvic pain, fever, vaginal discharge & negative lerrheic test. What is your diagnosis?
Correct Answer : B
PID is an infection of the female reproductive organs, It often presents with the above mentioned fearures.The negative Lerrheic test further supports the diagnosis of PID.
Q.98. During postpartum lady developed baby blues. Besides psychotherapy what will you advice?
Correct Answer : A
Baby blues is a common condition that can affect many women after childbirth. It is characterized by mood swings, crying spells, anxiety, and fatigue. While it usually resolves on its own within a few weeks, providing support and reassurance can alleviate symptoms.
Q.99. A pregnant lady presents with cystitis. Which of the following drug is contraindicated in her case?
Correct Answer : C
Fluroquinolones are contraindicated during pregnancy as they have the potential to cause cartilage damage and joint problems in the developing fetus.
Q.100. All of the following drugs are contraindicated in breast feeding, except?
Correct Answer : D
Ampicillin is generally considered safe to use during breastfeeding. It has a low level of excretion in breast milk and a low risk of causing side effects in infants.
Q.101. What is the initial evaluation in couples for infertility?
Correct Answer : B
Semen analysis is the initial evluation for couples with infertility as it assesses the quality and quantity of sperm, which are essential for fertilization.
Incorrect options-
- Temperature chart- It can help keep a track of ovulation, but can't evaluate fertility factors in both partners.
- Refer to reproductive clinic- not the initial step.
- Ovum analysis- more invasive and is therefore not the first step.
Q.102. What is the most common cause of male infertility?
Correct Answer : A
Primary hypogonadism is the most common cause of male infertility. In this condition the testicles fail to produce sufficient testosterone or sperm due to problems within the testicles themselves.
Incorrect options-
- Secondary hypogonadism- faliure of pituitary gland to signal testicles to produce sufficent testosterone, it can cause infertility but is less common than primary hypogonadism.
- Ejaculation obstruction- issue in the passageway of sperms which can cause infertility, but is less common compared to primary hypogonadism.
- Premature ejaculation- a sexual dysfunction in which ejaculation occurs rapidly, not considered a primary cause of male infertility.
Q.103. A pregnant woman never did an ANC checkup before, her baby was born with hepatosplenomegaly and jaundice. What is the cause?
Correct Answer : B
CMV is a common virus that can cause a range of symptoms including hepatosplenomegaly and jaundice, in newborns. CMV shows vertical transmission during pregnancy, which can lead to serious health issues in the baby.
Newborn complications caused by CMV-
- Hearing loss [most common long term complication]
- Partial or comlete vision loss
- Developmental delays
- Microcephaly
- Seizures
- Low birth weight
Other options can lead to birth defects but do not manifest as hepatosplenomegaly and jaundice of the newborn.
Q.104. A patient presents with thin cervix and scanty cervical mucus. How would you treat her?
Correct Answer : A
A thin cervical thickness and scanty cervical mucus often indicate low estrogen levels. Oestrogen injection can help make the cervical mucus thick and improve its quality, making it more conducive for sperm passage and fertilization.
Q.105. A pregnant woman 6 days post C-section, has a h/o blood stained discharge from her sutures from abdomen. What is the diagnosis?
Correct Answer : A
fascial dehiscence is defined as sepration of layers of surgical wound. Blood-stained discharge 6 days post C-section strongly suggests this condition. If a fascial dehiscence is observed, the patient should be taken immediately to the operating room where the wound can be opened, debrided, and reclosed in a sterile environment.
Incorrect options-
- Wound infection is suggested when excessive purulent discharge from the wound is present.
- Bacterial infection does not commonly present with blood stained discharge.
- Lochia is the normal vaginal discharge after childbirth and is unrelated to the abdominal sutures.
Q.106. A pregnant female in her third trimester presents with vaginal bleeding & abdominal pain. What is the first investigation you will perform?
Correct Answer : A
USG is the initial investigation of coice in this scenario.
Q.107. A 16 weeks pregnant woman complains of polydipsia & polyuria. Fasting sugar is 6.8. What is the diagnosis?
Correct Answer : A
IFG: (6.1-7.0 mmol), IGT: (7.8-11.1 mmol/l, 2h after 75g )
- First step: One-hour 50-g glucose challenge test; venous plasma glucose is measured one hour later (at 24–28 weeks). Values > 140 mg/dL are considered abnormal.
- Next step: Confirm with an oral three-hour (100-g) glucose tolerance test showing any two of the following: fasting > 95 mg/dL; one hour > 180 mg/ dL; two hours > 155 mg/dL; three hours > 140 mg/dL.
Q.108. A female presents with breech presentation at 34 weeks, what is the treatment option?
Correct Answer : C
At 34 weeks, it's still relatively early. The baby still has a chance of turning to cephalic presentation. Waiting and monitoring is the usual approach in this stage.
Q.109. Post-partum hemorrhage happens more commonly with which of the following?
Correct Answer : A
Multiple pregnancies increase the risk of PPH due to the following factors-
- Overstretching of uterus to accomodate multiple babies which in turn decreases the contractility.
- Multiple pregnancies have an increased chance of having placental abnormalities like placenta previa, accreta, which can cause excessive bleeding.
Other options given are not a strong risk factor for PPH.
Q.110. A 34 weeks GA lady presented with painful vaginal bleeding of amount more than that of her normal cycle. On examination uterus contracts every 4 min, bulged membrane, the cervix is 3 cm dilated, fetus is in a high transverse lie and the placenta is on the posterior fundus. US showed translucency behind the placenta and the CTG showed FHR of 170, what is the best line of management?
Correct Answer : A
The given scenario is suggestive of ''Abruptio placenta''. It is a condition where the placenta prematurely seprates from the uterine wall and can lead to heavy bleeding, fetal distress and even death of baby and mother.
Immediate C-section is the standard treatment of this condition.
Incorrect options-
- Gice oxytocin- it is used to induce or augment labor, but its contraindicated in placental abruption as it can lead to increased uterine bleeding by further contracting the uterus.
- Rupture of membrane- not an appropriate management.
- Obsereve- not an appropriate step in this scenario.
Q.111. Post D&C the most common site of perforation is?
Correct Answer : A
Fundus is the thinnest and most vulnerable part of the uterus, especially in situations where excessive force is applied during the procedure.
Q.112. A 34-year-old female has HIV, with pap smear negative. What is true about cervical cancer screening?
Correct Answer : B
PAP screening should begin within three years of the onset of sexual activity or at the age of 21 in a patient with an uncertain history of sexual activity. HIV+ patients should be screened every six months during their first year of diagnosis and then yearly if the initial tests are negative.
Q.113. A primi at 35 weeks of gestation presents with pre-eclampsia, her BP is high with ankle edema. What's the next best step?
Correct Answer : D
Preeclampsia is a serious pregnancy condition characterized by high blood pressure and other signs of organ damage, such as protrein in the urine, or edema.
Hospitalization and monitoring are crucial for women with preeclampsia so that they do not progress towards more sever condition like eclampsia.
Incorrect options-
- Diuretics- children of mothers with hypertension in pregnancy plus diuretic treatment in the third trimester were at significantly increased risk of developing schizophrenia.
- Low salt diet- may be helpful in lowering the BP, but its not the primary intervention.
- Labetalol is contraindicated in pregnancy.
Aim for delivery when the pregnancy is at term.
Q.114. A married couple is trying to have baby since last 3 months. What will you advice?
Correct Answer : A
If a couple has been trying for less than a year and has no underlying fertility issues, the advice is usually keep trying.
Even after having tried for one year yet no results, then refer to a specialist.
Q.115. What is the most common complication after hysterectomy?
Correct Answer : C
Complications of hysterectomy-
- Hemorrhage: Average intraoperative blood loss is 400 mL. Excessive bleeding complicates 1 to 3 percent of hysterectomy.
- Infection: Approximately one-third of women undergoing abdominal hysterectomy without antibiotic prophylaxis develop postoperative fever.
- Thromboembolic disease: in 0.2 and 2.4 percent.
- Ureteral injuries
Q.116. A female at 16 weeks of gestation presented with (++) glycosuria, FBS 4.4, 1 hours PB= 8, 2 hours PB= 7.2. What is the diagnosis?
Correct Answer : A
Renal glucosuria is the excretion of glucose in the urine in detectable amounts at normal blood glucose concentrations or in the absence of hyperglycemia. In general, renal glucosuria is a benign condition and does not require any specific therapy. Glycosuria may be associated with tubular disorders such as Fanconi syndrome, cystinosis, Wilson disease, hereditary tyrosinemia, or oculocerebrorenal syndrome (Lowe syndrome).
Q.117. What is the first sign of magnesium sulfate toxicity?
Correct Answer : A
Loss of deep tendon reflexes [ like knee jerk reflex] is one of the earliest signs of magnesium sulfate toxicity.
Incorrect options-
- Hypotension- it is a potential side-effect of magnesium sulfate, but is usually not the first sign of toxicity.
- Flaccid paralyisis- it is a more severe sign of magnesium sulfate toxicity and occurs later.
- Respiratory faliure- serious but not initial complication of magnesium sulfate toxicity.
Q.118. What is the contraindication to stop preterm delivery?
Correct Answer : D
[Preterm delivery refers to the delivery of baby before completing 37 weeks of gestation].
- Amniochorionitis- it is an infection of amniotic fluid surrounding the baby. It can lead to serious complications for the baby and requires prompt delivery.
- Placenta abruption- It is a condition where the placenta prematurely seprates from the wall of the uterus. It can cause severe bleeding and requires immediate delivery.
Q.119. What is the most common site of gonococcal infection in females?
Correct Answer : A
- The first place this bacterium infects is usually the columnar epithelium of the urethra and endo cervix.
- Non-genital sites in which it thrives are in the rectum, oropharynx, and the conjunctivae.
- The vulva and vagina are usually spared because they are lined by stratified epithelial cells.
Q.120. Before you start instrumental delivery, what is important to check?
Correct Answer : B
Instrumental delivery refers to the use of tools like forceps or vaccum to assist in delivery of the baby.
CPD is a clinical condition in which the baby's head is too large to fit through the mother's pelvis. Therefore before attemting instrumental delivery its crucial to rule out CPD.
Incorrect options-
- Face presentation- though it's an abnormal position but instrumental delivery is not contraindicated, given the baby is health and the mother's pelvis is adequate.
- Breech presentation- instrumental delivery is generally not recommended.
- Cord prolapse- it's a medical emergency and requies immediate delivery by C-section.
Q.121. A pregnant female develops lesions on the vulva and vagina and which was diagnosed as genital herpes, what should be included in her future health care management?
Correct Answer : B
HSV in pregnant is treated by: oral acyclovir 400 mg TID for 5-7 days.
[If HSV was present at the time of labor: c-section].
Q.122. Regarding postpartum depression, what is the most appropriate intervention to reduce the symptoms?
Correct Answer : A
Postpartum depression is a serious condition that affects new mother's. It is characterized by feeling of sadness, anxiety, and fatigue. In many cases treatment with therapy is sufficent but it is important to know that involving family in this process can be benificial.
Q.123. A pregnant lady delivered a baby with anencephaly. What is the chance of having birth occurrence of neural tube defect in next pregnancy?
Correct Answer : B
Neural tube defects arr birth defects that happen when a baby's spinal cord or brain doesn't form properly during the first month of pregnancy. Anencephaly is a severe form of NTD in which a major portion of brain and skull are missing.The recurrence risk is estimated to be around 2%.
Q.124. A female patient is in her 3rd week postpartum. She says to the physician that she frequently visualizes snakes crawling to her baby’s bed. She knows that it is impossible but she cannot remove the idea from her head. She says she wakes up around 50 times at night to check her baby. This problem prevents her from getting good sleep and it started to affect her marriage. What is this condition she is experiencing?
Correct Answer : C
Postpartum psychosis is a severe mental health condition that occurs in femalses after child birth.
- Onset- sudden,within days to weeks after delivery.
Features of postpartum psychosis-
- Severe mood change- mania or depression
- Hallucination [auditory/visual]
- Delusions [false beliefs often about baby]
- confusion and disorganized thinking.
- Agitation, insomnia, and catatonia
Q.125. In occipito-posterior malpositioning of the fetal head, all of the following are true except?
Correct Answer : D
In a normal labor the baby's head flexes to fit through the birth canal. However, in OP position flexion can hinder rotation to the anterior position.
Q.126. What is true regarding weight gain in pregnancy?
Correct Answer : A
During pregnancy, the calorie requirement increases by 300-500 kcal/day.
Q.127. A 25-year-old female patient has amenorrhea since 2 years, her prolactin level is 400 ng/ml. What is the probability to have pituitary prolactin secreting adenoma?
Correct Answer : C
Prolactin levels in excess of 200 ng/mL are not observed except in the case of prolactin-secreting pituitary adenoma (prolactinoma).
In 50 % of those having high prolactin levels there is radiological changes in the sella turcica.
Q.128. Which of the following is not compatible with head engagement?
Correct Answer : C
Head engagement - it is defined as when the widest part [ususally the bi-parietal-diameter] has passed through the pelvic inlet into the pelvic cavity.
If 3/5 of the baby's head is still palpable, it indicates that the head has not yet engaged in the pelvis.
Incorrect options-
- Zero station- presenting part at the level of ishial spine, which is a sign of engagement.
- Crowning of the head- widest part of the baby's head is visible at vaginal opening, signifies engagement.
- BPD at ischial spine- hallmark of head engagement.
Q.129. A female who has recently inserted IUCD comes with watery brownish vaginal discharge & abdominal pain. What is the most likely diagnosis?
Correct Answer : C
After the insertion of IUCD there is a slight increase in the chance of suffering from bacterial vaginosis. This is because the insertion procedure can disrupt the normal balance of bacteria in the vagina.
Symptoms-
- Thin, gray, or white discharge.
- Itching or burning sensation.
- Abdominal discomfort or pain.
Q.130. A young pregnant lady (Primigravida), at 32 weeks of gestation came to you with c/o lower limbs and swelling since two weeks duration. She went to another hospital, where she was prescribed thiazides & loop diuretics. O/E- BP: 120/70, mild edema, urine dipstick: -ve and other parameters are normal. What is the best action?
Correct Answer : C
Edema of lower limbs during pregnancy is quiet common, especially towards the later stages. However, excessive edema may be indicative of preeclampsia.
Loop diuretics have the tendency to cause hypovolemia, electrolyte imbalance which can have adverse effect on both mother and fetus.So, the best management is to stop both diuretics and at the same time evaluate the cause of the edema.
Q.131. A female patient presents with hiatal hernia, which of the following is correct regarding this condition?
Correct Answer : A
During pregnancy hormanal changes and the increased pressure that is exerted by the growing fetus can lead to worsening of the symptoms of hiatal hernia.
Q.132. What is an absolute contraindication of OCP?
Correct Answer : A
Both history of DVT and breast cancer are absolute contraindication, but in history DVT is more accurate.
Q.133. For placenta previa all the mentioned statements are true, except?
Correct Answer : B
Increasing urerine tone in a patient with placenta previa is a significant concern, as it is indicative of onset of labor or placental abruption both of which are serious complications.
Q.134. Which of the following statement is true regarding postpartum psychosis?
Correct Answer : A
One of the concerning risks of postpartum psychosis is the risk of recurrence in subsequent pregnancies.Women who have experienced psychosis once are at a higher risk of experiencing it again in future pregnancies.
Q.135. A 38-year-old female came to you at your clinic and her pap smear report was unsatisfactory for evaluation. What is the next best action?
Correct Answer : D
If the Pap smear is unsatisfactory due to inadequate sampling or other obscuring factors like, inflammation, and there are no other risk factors, it's reasonable to repeat the test in a year. This allows for potential issues to be resolved and a more accurate evaluation to be performed
Q.136. A primigravida with 8 weeks pregnancy came to you with nausea & vomiting. Which feature of the following suggests hyperemesis gravidarum?
Correct Answer : A
Laboratory findings include-
- Ketonuria
- Increased urine specific gravity
- Elevated hematocrit and BUN level
- Hyponatremia ,Hypokalemia , Hypochloremia
- Metabolic alkalosis.
Q.137. A pregnant women G4P3+1, at 10 weeks of gestational age came to you with IUCD inserted & the string is out from os. What is the most important measure?
Correct Answer : B
In this scenario, the most important measure is to have the IUD removed by an OB/GYN.
- Even if the IUD is partially expelled, it increases the risk of infection, miscarraige, or premature labor. Therefore it's crucial to remove it to minimize these risks.
Q.138. When will the pregnancy test come positive?
Correct Answer : D
Home pregnancy tests detect the presence of the hormone hCG in urine. After implantation the, hCG levels start to rise rapidly. Most of the home pregnancy test kits are designed to detect hCG levels around 1 week before your expected period.
Q.139. A young lady presents with pelvic pain and menorrhagia, examination showed uterine mass. What is the diagnosis?
Correct Answer : A
Uterine fiboids are benign tumors that grow in the uterus and are a common cause of heavy menstrual bleeding and pelvic pain. The precense of uterine mass further supports the diagnosis.
Incorrect options-
- Adenomyosis- can cause mennorhagia, but do not present with abdominal mass.
- Endometriosis- can cause pelvic pain, irregular bleeding but not associated with abdominal mass.
- PID- can cause pelvic pain, irregular bleeding, but is not associated with abdominal mass.
Q.140. A 35-year-old G4 P2+1, presents with history of irregular heavy bleeding since 1 year. O/E WNL, what is the most diagnosis?
Correct Answer : D
DUB is the most likely diagnosis in this scenario. It refers to abnormal uterine bleeding that occurs in the abscence of any identifiable underlying medical causes.
Q.141. A 20-year-old married lady presented with history of left lower abdominal pain & amenorrhea for 6 weeks. What is the most appropriate investigation to rule out ectopic pregnancy?
Correct Answer : C
Pelvic ultrasound is the preferred initial diagnostic tool for ectopic pregnancy dur to its high specificity and senstivity.
Q.142. A 45-year-old female complains of itching in genitalia, she is afebrile, -ve PMH, living happily with her husband since 20 years. On examination no abdominal tenderness, redness of vagina , mild gray discharge +, no history of UTI or pyelonephritis. What is the most probable diagnosis?
Correct Answer : A
Types of vaginitis:
- Bacterial Vaginosis -->Gardnerella
- Vaginal candidiasis --> Candida
- Trichomoniasis -->Trichomonas vaginitis.
Q.143. A female patient presents with thick vaginal discharge, no color, no itching, vaginal examination was normal on speculum. PH: 4, what is the diagnosis?
Correct Answer : A
This refers to normal, healthy vaginal discharge produced by the body. It can vary in consistency and amount depending upon the hormonal cycle.
Q.144. A pregnant woman presents with a mass in her mouth, bleeding when brushing her teeth. On examination, a mass of 3x2 cm was found. What is the diagnosis?
Correct Answer : C
Pyogenic granuloma is a benign vascular tumor that can grow during pregnancy, it is considered a pregnancy tumor because of its emergence in the mouth area.
Q.145. What is most common cause of death during the first trimester?
Correct Answer : A
Ectopic pregnancy is the most common cause of pregnancy-related death during the first trimester. In this condition, the fertilized egg implants outside the uterus, typically in the fallopian tube. This can lead to severe internal bleeding and potentially life-threatening complications if not diagnosed and treated promptly.
Q.146. A female presents with monilial vaginal discharge. What is the treatment?
Correct Answer : A
Monilial vaginal discharge, also known as candida vulvovaginitis, is a yeast infection caused by overgrowth of the fungus Candida albicans. Miconazole is an antifungal medication that comen in various forms, including cream. Using it for 7 days effectively treats the infection by causing cell wall disruption.
Q.147. A female patient presents with itching in the vagina associated with the vaginal discharge, PH:5 , no trichomoniasis infection, pseudo hyphae were found on culture. What is the most likely diagnosis?
Correct Answer : B
According to the given scenario-
- The vaginal pH is 5 [acidic] which is the ideal environment for candida to thrive.
- Itching and discharge are common symptoms of yeast infection.
- Pseudo-hyphae on culture strongly supports the diagnosis of candida albicans.
Q.148. Hyperprolactinemia is associated with all of the following, except?
Correct Answer : C
Pregnancy, breastfeeding, mental stress, sleep, hypothyroidism, and use of prescription drugs are the most common causes of hyperprolactinemia.
In men, the most common symptoms of hyperprolactinemia are - decreased libido, erectile dysfunction, infertility, and gynecomastia.
Q.149. All are true about ectopic pregnancy, except?
Correct Answer : A
While ectopic pregnancy can occur in the ovary [about 0.5-1% cases], the statement that it accounts for 20% is incorrect. The most common site of ectopic pregnancy is fallopian tube.
Sites-
- Isthmus [10-12%]
- Fimbrial end [5%]
- Interstitial/ cornual [2-4%]
- Ovary [0.5-1%]
- Abdomen [<1%]
- Cervix [rare <1%]
Q.150. Non-hormonal treatment for post-menopausal flushing?
Correct Answer : A
Paroxetine is a SSRI antidepressant that is effective in treating hot flashes associated with menopause. It works by regulating serotonin levels in the brain, which can reduce the frequency and severity of hot flashes.
Q.151. Which of the following statement is true regarding obstructed labor?
Correct Answer : C
Obstructed labor occurs when the passage of baby is hampered through the birth canal due to physical blockage. This can lead to prolonged labor. Caput succedaneum and excessive molding of the baby's head are common signs of obstructed labor. Thses indicate that the baby's head has been pressing against the cervix for an extended period causing pressure and deformation.
Q.152. A primigravida presents with whitish discharge, the microscopic finding showed pseudo hyphae. What is the treatment?
Correct Answer : A
The presence of pseudo hyphae in the discharge is strongly suggestive of infection by Candida albicans.
- Miconazole cream is an antifungal medication that is effective against Candida.
Q.153. A pregnant teacher in her 20 weeks of pregnancy reported that 2 of her students had developed meningitis. What is the prophylactic treatment for her?
Correct Answer : D
Rifampicin is the recommended treatment for prophylaxis of close contacts of individuals with meningococcal meningitis.
[Ciprofloxacin is contraindicated in pregnancy].
Q.154. A patient presents with h/o cervicitis + strawberry cervix + mucopurulent yellow discharge. The cervix is eroded + friable. What is the diagnosis?
Correct Answer : A
Presence of ''strawberry cervix'' is highly suggestive of Trichomonas vaginalis infection.Trichomonas vaginalis is a protozoan that causes STI.
[strawberry cervix- a cervix with small, red petechiae]
Incorrect options-
- STIs like chlamydia and gonorrhea can cause discharge and cervicitis but lack the characterstic feature of strawberry cervix appearance.
- HSV causes genital ulcers and dischrge but not strawberry cervix.
Q.155. All of the following are causes of intrauterine growth restriction (IUGR), except?
Correct Answer : D
HSV 2 while a sexually transmitted infection, it is primarily associated with genital herpes and is not a typical cause of IUGR.
Other options-
Toxoplasmosis, CMV, and rubella can cross the placenta and cause IUGR in the developing fetus. They can interfere with the proper growth and development of fetus.
Q.156. A pregnant lady at 28 weeks of gestation presents with Chlamydia infection. What is the treatment?
Correct Answer : B
Erythromycin, 500 mg four times daily for seven days, is the treatment of choice during pregnancy & lactation.
Q.157. What is the management of post-partum hemorrhage?
Correct Answer : A
Oxytocin is the first-line drug for the management of PPH. It stimulates uterine contractions, which help compress the blood vessels and reduce bleeding. It is routinely used in active management of the third stage of labor to prevent PPH.
Misoprostol, a prostaglandin, is an alternative to oxytocin particularly in low-resource settings where oxytocin might not be available, it is considered a second- line option.
Incorrect options-
- Warfarin- is an anticoagulant, it would worsen the bleeding.
- Aspirin - it is also an antiplatelet and anticoagulant agent and will increase the risk of bleeding.
Q.158. A 38 weeks pregnant lady presented with placenta previa marginal, with mild bleeding, cervix is 2cm dilated. How to manage?
Correct Answer : B
In the given scenarion the placenta is located close to the cervix but not completely covering the cervical opening. In case of mild bleeding and a favourable cervical dilation spontaneous vaginal delivery can be considered.
This decision would depend on several factors, including the severity of bleeding, fetal well-being, and the mother's overall health.
Q.159. Which heart condition is tolerable during pregnancy?
Correct Answer : C
While severe mitral regurgitation is a serious condition, it can often be managed during pregnancy, with careful monitoring and medical treatment.
Q.160. A female patient presents with hirsutism, obesity, and infertility. USG shows multiple ovarian follicles. What is the diagnosis?
Correct Answer : D
Stein-Leventhal syndrome is the other name for PCOS, and is characterized by the triad of hirsuitism, obesity, and infertility. The precense of multiple ovarian follicle on ultrasound is a hallmark feature of PCOS.
Q.161. A young female presents with few tear like vesicles on rose red colored base of vulva, and has pain. What is the cause?
Correct Answer : B
The most likely cause of the described symptoms is HSV. Genital herpes often presents with clusters of small, painful blisters on the genitals. The characterstick rose-red base is also typical feature of herpes.
Incorrect options-
- Syphills- painless chancre at site of infection
- Chancroid- painful soft sore on the genitals
- HPV- painless, cauliflower like growths
Q.162. A pregnant lady underwent USG which showed anterolateral placenta. On vaginal examination - examiner’s finger can’t reach the placenta. What is the diagnosis?
Correct Answer : D
Placenta previa occurs when the placenta partially or completely covers the cervical opening. In this case the per vaginal examination reveals that the placenta is not covering the cervix as the examiner's finger cannot reach the placenta.
Anterolateral placenta means that the placenta is located on the anterior and towards one lateral side. This is the normal position of placenta.
Q.163. A 52-year-old women complaints of loss of libido, dry vagina, reduced concentration, and weight gain since 10 months, this has affected marital life. Which of the following will you give her?
Correct Answer : C
The women in the given scenario is experiencing symptoms of menopause. While HRT with estrogen and progesterone can help alleviate few symptoms, it is not the first-line treatment for all menopausal women.
Fluoxetine is an antidepressant medication that is effective in treating menopausal symptoms by increasing the level of serotonin in the brain, which can also help to improve mood and reduce anxiety.
Q.164. What is the most dangerous condition in menopause?
Correct Answer : A
Ovarian cancer is also called the serial killer as it develops without any noticeable symptoms in its early stages. By the time symptoms appear the cancer may have already spread, making it difficult to treat.
Q.165. A 20-year-old woman who is sexually active suffers from pain during intercourse. On urine analysis, gram negative diplococci, intracellular were seen. What is the diagnosis?
Correct Answer : A
The scenario is a classic presentation of of gonorrhea, a sexually transmitted infection caused by bacterium Neisseria gonorrhoeae.
The prescnce of gram-negative diplococci within cells strongly suggests diagnosis of gonococcal infection.
Q.166. What is the most lethal infection in a pregnant woman?
Correct Answer : A
Toxoplasmosis can be transmitted to the fetus through the placenta. In pregnant women especially during the first trimester, toxoplasmosis can lead to severe complications of the fetus such as abortion, still birth, and severe birth defects.
Incorrect options-
- HIV- can be managed by ART , reducving the risk of transmission and severe complication.
- Rubella- can lead to congenital rubella syndrome, but is preventable by vaccine.
- Measeles- can increase risk of abortion, premature birth, and low birth but is preventable by vaccination.
Q.167. A 32-year-old has 2 children, her pap smear showed atypical squamous cells. What is the next step?
Correct Answer : C
Colonoscopy is a procedure in whcih a special magnifying instrument with a light source attached to it is used to examine the cervix and vagina for any abnormalities. It is the next step in evaluating atypical squamous cells detected on PAP smear.
Q.168. A patient complains of dysmenorrhea + amenorrhea. She also has a h/o Infertility. What is the diagnosis?
Correct Answer : A
Endometriosis is a condition where endometrial tissue grows in places other than the uterine cavity, such as ovaries, fallopian tubes, or even other organs. This tissue at abnormal sites can lead to a variety of symptoms, including-
Dysmenorrhea, amenorrhea, and infertility.
Q.169. Which of the following oral contraceptive drugs causes hyperkalemia?
Correct Answer : B
It is an opioid medication used to treat pain .It causes hyperkalemia as a side effect.Therefore it is the only contraceptive drug listed that can cause hyperkalemia.
Q.170. A woman in her early pregnancy comes to your clinic, which of the following is most beneficial to do?
Correct Answer : C
In early pregnancy USG is the most accurate test to confirm pregnancy. Along with confirmation it gives infomation regarding various other parameters related to the fetus including- estimated gestational age, EDD, fetal viability ,etc.
Incorrect options-
- CBC - gives info about the overall health of mother
- Urine pregnancy test- It can confirm pregnancy but does not provide information about fetal development
- MRI- not typically done
Q.171. Which of the following is true about premenstrual depression syndrome?
Correct Answer : B
Premenstrual syndrome: is defined as a symptoms complex of physiological and emotional symptoms severe enough to interfere with everyday life and occur cyclically during the luteal phase of menses.
Premenstrual dysphoric disorder: is a severe form of premenstrual syndrome characterized by severe recurrent depressive and anxiety symptoms with premenstrual (luteal phase) onset that remit a few days after the start of menses.
Q.172. A female on OCPs comes to your clinic with complains of skin changes on the face. What is the diagnosis?

Correct Answer : B
Melasma is a common skin condition that causes brown patches on the face, often triggered by hormonal changes, sun exposure or certain medications
Q.173. Which of the following is true regarding spontaneous abortion?
Correct Answer : C
Cervical assessment is a crucial part of managing a woman who has expreienced a spontaneous abortion. It helps to determine the cause of abortion and assess the risk of future complications.
Incorrect options-
- 30-40% of pregnancies end with of miscarrriage- not accurate
- Most of them happen in the seconfd trimester- most of them acually occur in first trimester.
- Most of them happen in third trimester- less common in third trimester.
Q.174. Which of the following is true about fetal alcohol syndrome?
Correct Answer : B
FAS can lead to LBW and small head size.
- Facial abnormalities- smooth philtrum, thin upper lip and small eye opening.
- CNS abnormalities- intellectual disability, poor coordination, and behavioural issues.
Q.175. Which of the following statement is true regarding secondary amenorrhea?
Correct Answer : B
Sheehan's syndrome is a condition that occurs due to severe blood loss or low blood pressure during or after childbirth, leading to pituitary gland damage. Since the pituitary glad is the powerhouse of hormone regulation, its damage can result in secondary amenorrhea.
Incorrect options-
- Due to gonadal agenesis- related to primary amenorrhea.
- It's always pathological- It can be due to non- pathological causes like pregnancy, stress, or lactation.
- Causes infertility- it is not directly linked to causing infertility.
Q.176. A patient presents with herpes in vagina, which of the following is true?
Correct Answer : B
Herpes infection during pregnancy can pose risk to newborn, such as neonatal herpes.If the mother has an active infection around the time of delivery, a C-section is recommended to prevent the virus from being transmitted to the baby during delivery.
Q.177. A female with atypical squamous cells of undetermined significance (ASCUS) on pap smear, was started on a 30 day treatment with estrogen & was told to come back after 1 month. She still shows +ve on pap smear. What's the next step to be performed?
Correct Answer : B
ASCUS on PAP smear is indicative of abnormal cells in cervix. If any suspicious areas are found during coloscopy, a biopsy can be taken to determine the malignant potential of the cells.
Incorrect options-
- Biopsy- performed after colposcopy.
- VDRL - done for screening of STD's.
- CBC- not done for ASCUS.
Q.178. A female presented with complain of post-coital bleeding, a cervical mass was found on examination. What is the next step?
Correct Answer : C
Post-coital bleeding and precense of cervical mass are strong indicators of malignant potential.
Direct biopsy is the most appropriate step to obtain tissue sample and perform further evaluation. This will help determine the nature of the mass.
Incorrect options-
- Cone biopsy- it's an extensive procedure and is usually done after diagnosis of cervical cancer that has been confirmed through direct biopsy.
- Pap smear- it's a screening test for cervical cancer.
- Colposcopy- it can help identify abnormal areas but is not a diagnostic test.
Q.179. What is the most common cause of ectopic pregnancy?
Correct Answer : A
Salpingitis is the inflammation of the fallopian tubes often caused by infections like chlamydia or gonorrhea. Scarring due to inflammation caused by these infective agents leads to the blokage of fallopian tubes, preventing the fertilized egg from reaching the uterine cavity leading to ectopic pregnancy.
Q.180. A pregnant woman with uterine fibroid has no symptoms, only abdominal pain. USG showed live fetus. What is the appropriate action to take?
Correct Answer : C
The precence of a uterine fibroid in a female during pregnancy is not always indicative of an immediate intervention, especially if the women is asymptomatic. In the above mentioned scenario the main concern for which the female presented to the clinic is abdominal pain. Pain management methods such as over-the-counter pain relievers or stronger medications would be the appropriate first-step in her management.
Q.181. What's the drug used in seizures of eclamptic origin?
Correct Answer : A
Magnesium- sulfate is the first- line drug used to treat seizures associated with eclampsia. it stabalizes the neuronal membranes and acts as a CNS depressant.
[ Remember-in eclampsia it ''treats seizure'', in severe preeclampsia it ''prevents seizure''].
Q.182. Pregnant woman previously had DVT. What prophylaxis will you give her now?
Correct Answer : D
Enoxaparin is a LMWH which is considered the preferred anticoagulant for thromboprophylaxis in pregnant women with history of DVT.
They have a more controlled and predictive anticoagulant effect and lower risk of bleeding complications which are observed in unfractioned heparin or warfarin.
Q.183. A pregnant female has HIV, what is the most accurate statement regarding risk of transmission of HIV to the baby?
Correct Answer : D
Breastfeeding is a significant route of HIV transmission from mother to child.
Q.184. A pregnant woman on iron supplementation throughout her pregnancy for anemia, now comes complaining of weakness and easy fatigability. Her hemoglobin is 7 mg/dl, MCV 60. What is the diagnosis?
Correct Answer : D
The patient's blood report is indicative of anaemia. However, despite taking iron supplementation, the anaemia persists which is suggestive of anemia due to any other cause. So, it can be due to Beta thalassemia trait which can also present with low Hb and microcytosis.
Q.185. A pregnant lady developed sudden left leg swelling. What's the best management?
Correct Answer : D
- Duplex scan- it is crucial for diagnosis as it visualizes the blood flow in the veins.
- Rest- helps in reducing inflammation and discomfort.
- Heparin- prevents formation of blood clot.
Q.186. A female developed low BP next day after CS. What is the management?
Correct Answer : A
Postpartum hypotension should bring ones attention towards ruiling out PE by imaging and administering I/V heparin that helps in preventing the blood clot from increasing in size or forming new clots.
Q.187. A 14-year-old female complains of irregular bleeding, examination shows normal sexual character, normal vagina and cervix. What to tell her?
Correct Answer : A
Irregular bleeding in a 14-year-old is often due to anovulatory cycles, which is common in first few years of menarche.
Q.188. A perimenopausal female gives history of menorrhagia since last 3 months, her Hb is 8. What is the first action that you will take?
Correct Answer : B
Menorrhagia can lead to anaemia, in case of severe symptomatic anaemia the first priority is to stabalize the patients condition. Hospitalization for blood transfusion is crucial to quickly correct anaemia.
Q.189. A patient 2-3 days before menstruation presents with depressed mood that disappears by 2-3 days after the beginning of menstruation. What is the diagnosis?
Correct Answer : A
PMS is a common condition in women characterized by physical and emotional symptoms that occur during the luteal phase of the menstrual cycle. The symptoms aggrevate a week before the mensturation begins and alleviate after the onset of bleeding.
Q.190. Methylergonovine is used in which of the following?
Correct Answer : D
Methylergonovine is used in the following-
- Maternal HTN
- PPH prevention
- Migraine treatment
Q.191. A nulliparous woman came with menorrhagia, Hb is 10. What is the management?
Correct Answer : A
Estrogens usually controls severe acute bleeding quickly. However, when estrogens fail to control it, dilation and curettage [D&C], is sometimes necessary.
Q.192. An old female presents with itching of vulva. On examination cervix and vagina is pale and thin. No discharge. What is management?
Correct Answer : A
Most of the common menopause prescriptions include-
- Hormone replacement therapy or anti-depressants to minimize hot flashes.
- Fosamax or Actonel (non-hormonal medications) to reduce bone loss and reduce the risk of fractures.
- Selective estrogen receptor modulators (SERMs) mimic estrogen's beneficial effects on bone density.
- Vaginal estrogen, administered locally, to relieve vaginal dryness and discomfort during intercourse.
Q.193. Which is true regarding Trichomoniasis?
Correct Answer : C
Trichomoniasis is a sexually transmitted protozoal infection. It’s the most common curable sexually transmitted disease.It causes a yellow-green, malodorous, diffuse discharge in addition to dysuria, frequency, petechiae on the vagina and cervix, and irritated and tender vulva.
On microscopy-Saline (wet mount) will show motile flagellated organisms, WBCs, and inflammatory cells.
- Treatment -2 gm metronidazole single dose P.O. (same for pregnancy).
- Always treat the partner.
Q.194. A pregnant woman has allergy for Sulfa drugs & penicillin. Which drug is safe for her?
Correct Answer : A
Nitrofurantoin is an antibiotic that is generalyy considered safe to use during pregnancy. It has low risk of causing birth defects and other complications.
Q.195. A female presents with a negative pap smear. What should you advice her about pap smear?
Correct Answer : B
- Screening Pap smear: Starting at age 21 years or no more than 3 years after becoming sexually active
- Women > 30 years who have three consecutive normal tests screening (1 / 3 years).
- Screening should be discontinued for women > 60-70 years who have had 3 or more normal Pap smear.
Q.196. A pregnant woman presents with UTI. What is the best antibiotic if she has no allergy?
Correct Answer : A
Nitrofurantoin is generally considered safe to use during pregnancyfor treating UTI.
Q.197. A multiparous woman comes with 1 year history of stress incontinence. What is the treatment?
Correct Answer : B
Stress incontinence is the involuntary leakage of urine during activities that increase abdominal pressure. Kegel exercises strengthen the pelvic floor muscles, are first-line treatment for stress incontinence.
Q.198. Which of following increases during pregnancy?
Correct Answer : A
Tidal volume is the amount of air inhaled and exhaled during normal, quiet breathing. Increase in tidal volume helps to meet the increased oxygen demands of the mother and fetus.
Q.199. An old woman did a Pap smear which was negative, then after 7 years she did another Pap smear which showed Squamous metaplasia undifferentiated. What is your next step?
Correct Answer : C
Colposcopy is the most appropriate next step.
Q.200. A 62-year-old female complains of pruritis in pubic area with bloody discharge; she tried different treatment but no improvement. She then developed pea shaped mass in her labia. What is the diagnosis?
Correct Answer : A
Bartholin’s cyst is formed when Bartholin's gland is blocked, causing a fluid-filled cyst to develop. A Bartholin's cyst can be caused by an infection, inflammation, or physical blockage to the Bartholin's ducts.
- If infection sets in, it results in a Bartholin's abscess
if the infection is severe or repeated a surgical procedure known as marsupialization.
Q.201. What is the most common cause of precocious puberty?
Correct Answer : A
In most cases of precocious puberty, there's no identifiable cause, and the body starts producing hormones too early without a clear reason.
Other options are incorrect as they can cause production of hormones but are less common.
Q.202. A lactating woman 10 days after delivery developed fever, malaise, chills, tender left breast with hotness and a small nodule in upper outer quadrant with axillary LN. Leucocytic count was 14 *10/L. What is the diagnosis?
Correct Answer : B
The symptoms in the scenario are strongly suggestive of breast abscess, which is a collection of pus caused by bacterial infection in the breast tissue.
Incorrect options-
- Inflammatory breast disease- nodule and symptoms point towards abscess.
- Fibrocystic disease- benign condition causes breast lump but doesn't present with fever or infection symptoms.
- Post-partum breast changes- normal changes like engorgement don't cause fever, chills, or localized abscesses.
Q.203. What physiological change occurs during pregnancy?
Correct Answer : B
During pregnancy cardiac output rises by 50% to meet the increased blood flow that is needed by the placenta.
Incorrect options-
- Cardiac output decreases- this is wrong as the demand increases.
- Stroke volume decreases- it doesn't decrease significantly, it only compensates the increased cardiac output.
- Heart disease associated - pregnancy doesn't cause heart disease but can worsen existing conditions.
Q.204. Which of the following is not used in the prevention of preeclampsia with proteinuria & lower limb edema?
Correct Answer : B
Diuretics are not used in preeclampsia as they reduce the blood volume, potentially worsening the already compromised blood flow to the placenta. This can increase the risk of placental insufficiency and lead to fetal complications.
Q.205. A women complains of a non-fluctuant tender cyst on the vulva. She also complains of pain while coitus & walking. She was diagnosed to have Bartholin’s cyst. What is the treatment?
Correct Answer : A
A Bartholin's cyst forms when gland's duct becomes blocked, leading to fluid accumulation. If the cyst becomes painful or large, the best treatment is incision and drainage. This quick procedure creates a small oprning to release the fluid, offering immediate relief.
Incorrect options-
- Refer to surgery for exision- complete removal of gland is only considered when it is recurring.
- Reassurance is not appropriate if the cyst is painful or causing discomfort.
- Antibiotics are only used if the cyst in infected.
Q.206. A pregnant lady is in her 3rd trimester, she is diabetic and on insulin. She compliant to medication but has suffered from hyperglycemic attack. What is the common complication in fetus?
Correct Answer : B
When a diabetic mother has a hyperglycemic attack, the fetus is exposed to high levels of glucose through the placenta. In responce the fetal pancreas produce excess insulin. After the mothers glucose levels are corrected and drops, the baby's insulin remains high, resulting in neonatal hypoglycemia.
- Other options are incorrect as they do not correctly relate to the mechanism.
Q.207. A pregnant patient wants to take varicella vaccine. What will you advice her?
Correct Answer : A
The varicella vaccine is a live attenuated vaccine, which means that it contains a weak form of the live virus. It can have potential risk on the developing fetus during pregnancy. Live vaccine can theoritically cross placenta and infect the fetus.
Q.208. A pregnant lady with 2 months of gestation presented with excessive vomiting and generalized swelling of the body. She has increased Beta HCG, no fetal parts seen on U/S. BP is slightly raised. Size of uterus is more than period of gestation. What is the diagnosis?
Correct Answer : A
The combination of excessive vomiting, generalized swelling, increased Bets HCG levels, no fetal parts on USG, and a slightly raised BP are strongly suggestive towards the option of hydatidiform mole. This is a clinical condition characterized by abnormal proliferation and degeneration of trophoblastic tisssue in placenta, forming a mass of fluid- filled cysts instead of normal placenta.
Types of mole-
- Complete mole- no fetus present, placental tissue is completely abnormal typically resulting from fertilization of an empty egg by one or two sperm.
- Partial mole- fetal tissue or fetus may be present but is not viable, caused by fertilization of normal egg by two sperms leading to triplody.
Diagnosis-
- USG- ''snowstorm appearance'' or cluster of grapes apperance is seen.
- Beta-hCG levels- elevated
Q.209. A pregnant lady, 16 weeks of GA, on USG finds that the fetus is small for age. On P/E uterus size 12w. What is the diagnosis?
Correct Answer : D
This discrepancy between the GA and the uterine size is suggestive of gestational age discordance, which will be requiring further evaluation, but not fitting any related options.
Incorrect options-
- Choriocarcinoma- rare and agressive type of GTN that can develop after a hydatidiform mole or a normal pregnancy. It's unlikely in this scenario because the fetus is live, not a post-pregnancy condition.
- Hydatidiform mole/Molar pregnancy- these terms refer to abnormal placental growth where the fetus is usually absent. Since the woman has a small fetus this is not the case.
Q.210. A pregnant women at 36 weeks of gestation presents with vaginal bleeding since 2-3 hours, she has h/o 3 NVD. Which of the following is important to ask?
Correct Answer : B
In a late pregnancy, vaginal bleeding can be a concerning sign. One of the potential causes of vaginal bleeding during this age is placental abruption. This is a serious condition in which the placenta prematurely septrates from the utrine wall.
Intercourse can increase the risk of abruption. The movements and preassure during intercourse can sometime trigger the sepration. Therefore, inquiring about recent intercourse is crucial to assess the potential risk of abruptio placenta.
Incorrect options-
- Smoking- it's generally harmful in pregnancy but is less directly linked to triggering bleeding in late pregnancy.
- Alcohol consumption- not directly associated.
- Tobacco use- not as relevant to cause bleeding at this stage.
Q.211. Which of the following is not correct during management of labor?
Correct Answer : D
Enemas are no longer routinely recommended during labor. They can cause electrolyte imbalance and discomfort and offer no significant benifit in preventing infection.
Q.212. What requirements must be fulfilled before an instrumental delivery is performed?
Correct Answer : C
Instrumental deliveries can cause significant trauma to the cervix and the baby if performed without complete dilation.
Q.213. What is common in a vesicular mole?
Correct Answer : D
Theca-lutein cysts are functional ovarian cysts, that are frequently seen in association with molar pregnancies due to high levels of hCG.
Q.214. Which of the following test is mandatory for all pregnant women?
Correct Answer : D
All of the following tests are recommended as part of routine prenatal care for all pregnant women.
Routine prenatal investigations-
- CBC
- Blood grouping and Rh typing
- Urine analysis
- Blood glucose levels
- Thyroid function tests
- HIV, hepatitis, syphilis
- Rubella immunity status
Q.215. Which of the following suggests enormous ovarian cyst?
Correct Answer : D
This percussion finding is characterstick of an ovarian cyst, the central dullness is due to fluid-filled cyst, while the lateral tymanic note is caused by the gas filled intestines being pushed to the sides by the cyst.
Q.216. Perinatal asphyxia is caused by all, EXCEPT?
Correct Answer : B
This is a condition characterized by severe nausea and vomitting during pregnancy. While it can be debilitating for the mother, it does not directly cause fetal asphyxia.
All other options contribute in causing fetal asphyxia.
Q.217. A healthy 28 year old lady P1+0 presented to you with 6 months amenorrhea. What is the most likely cause for her amenorrhea?
Correct Answer : A
The most common cause of secondary amenorrhea in a female of reproductive age is pregnancy.
Q.218. What is the mechanism of action of contraceptive pills?
Correct Answer : D
Inhibition of midcycle gonadotropins then ovulation- This is the primary mechanism of how majority of COCs work. They contain synthetic versions of estrogen and progesterone. These hormones prevent the surge of LH and FSH, which are necessaary for ovulation. By inhibiting these hormones, COCs prevent the release of egg from the ovary.
Q.219. Dyspareunia is caused by all of the following, EXCEPT?
Correct Answer : D
Uterine prolapse typically causes pelvic preassure, urinary symptoms, or discomfort, but it is not a primary cause of painful intercourse.
Q.220. Anti D Immunoglobulin is not given to a pregnant lady in which of the following cases?
Correct Answer : B
Anti-D is routinely given to un-sensitized mothers at 28 and 34 weeks of gestation. Fetomaternal hemorrhage sensitizes susceptible mothers to develop anti-D antibodies (e.g. Birth, Miscarriage, abortion, amniocentesis, vaginal bleeding, external cephalic version etc)
Indications for Anti-D - The initial response to D antigen is slow sometimes taking as long as 6 months to develop (rising titers).
Q.221. Which of the following causes blockage of first stage labour pain?
Correct Answer : A
Lumbosacral afferent nerves carry sensory information towards the CNS. The lumbosacral plexus is involved in innervating the uterus. Blocking these afferent nerves will interrupt the transmission of pain signals fom the contracting uterus to the brain, providing pain relief during the first stage of labor.
Q.222. What is true regarding premenstrual tension?
Correct Answer : C
Premenstrual syndrome (PMS) is a recurrent luteal phase condition (2nd half of menses) characterized by physical, psychological, and behavioral changes of sufficient severity to result in deterioration of interpersonal relationships and normal activity.
The most common signs and symptoms associated with premenstrual syndrome include:
(1) Emotional and behavioral symptoms-
- Tension or anxiety
- Depressed mood Crying spells
- Mood swings and irritability or anger
- Appetite changes and food cravings
- Trouble falling asleep (insomnia)
- Social withdrawal Poor concentration
(2) Physical signs and symptoms-
- Joint or muscle pain
- Headache
- Fatigue
- Weight gain from fluid retention
- Abdominal bloating
- Breast tenderness
- Acne flare-ups
- Constipation or diarrhea
One study has shown that women with PMS typically consume more dairy products, refined sugar, and high-sodium foods than women without PMS. Therefore, avoidance of salt, caffeine, alcohol, chocolate, and/or simple carbohydrates may improve symptoms.
Q.223. If a pregnant woman is eating well balanced diet, which one of the following should be supplied?
Correct Answer : D
A well balanced diet typically provides all essential nutrients that a pregnant women needs, including calcium, phosphate, and vitamin C. Therefore, it is not necessary to supplement any of these nutrients specifically.
Q.224. All of the following are normal flora and should not be treated, EXCEPT?
Correct Answer : D
Normal flora refers to microorganisms that naturally live in and on our bodies.They generally do not cause harm and can even be benificial.
Trichomonas is a parasite, not part of the normal flora that is transmitted sexually. This infection is often treated with antibiotics.
Q.225. All the following drugs should be avoided in pregnancy, EXCEPT?
Correct Answer : C
Cephalexin [keflex]- This cephalosporin antibiotic is generally considered safe to use during pregnancy especially in later stages.
Incorrect options-
- Sodium valproate- this anticonvulsant drug is strongly related to birth defects [neural tube defects].
- Glibenclamide- this oral hypoglycemic drug can cross placenta and potentially harm the developing fetus.
- Septrin- this combination antibiotic can increase the risk of birth defects.
Q.226. Cord prolapse occurs in all EXCEPT?
Correct Answer : C
Oligohydramnios is a condition where there is too little amniotic fluid surrounding the fetus. While it increases the risk of certain complications, it does not directly lead to cord prolapse.
Q.227. Regarding breech presentation all are true, EXCEPT?
Correct Answer : A
By 36 weeks of pregnancy incidence of breech presentation are much lower than 22%.
Q.228. Signs and symptoms of normal pregnancy are all, EXCEPT?
Correct Answer : A
Hyperemesis is a severe form of morning sickness that leads to excessive nausea, vomitting, and weight loss. It is not considered a normal part of pregnancy.
Other options-
- Hegar sign: softening of the lower uterine segment.
- Chadwick's sign: bluish discoloration to the cervix and vaginal walls.
- Amenorrhea is the abscence of menstruation, a very common symptom of pregnancy.
Q.229. In twins all are true, EXCEPT?
Correct Answer : B
TTTS is a rare but serious condition that occurs in monochorionic twins where the twins share a single placenta. It is not common in dizygotic twins as they have their own placenta.
Q.230. In lactation all are true, EXCEPT?
Correct Answer : C
While staying hydrated is important for overall health and milk production, mild dehydration is unlikely to significantly decrease milk supply.
Q.231. Patient with postpartum haemorrhage & infertility, all can be found EXCEPT?
Correct Answer : A
Ballooning of the sella turcica is a sign of pituitary adenoma, a benign tumor of the pituitary gland. While pituitary issues can sometimes contribute to infertility, they are not typically associated with postpartum hemorrhage.
Q.232. Placenta previa, all are true EXCEPT?
Correct Answer : A
While placenta previa can cause bleeding, the amount of bleeding usually doesn't cause shock. Shock is a life-threatening condition where the tissue perfusion in not adequate due to decreased blood flow.
Q.233. In pelvic inflammatory disease, all are true, EXCEPT?
Correct Answer : B
Endometriosis is a clinical condition characterized by growth of endometrial tissue at sites other than the normal site in uterine cavity.While it can sometimes cause pelvic pain and infertility, it is a distinct condition from PID.
Q.234. All of the following are responsible for recurrent abortion , EXCEPT?
Correct Answer : D
Uncontrolled diabetes can increase the risk of pregnancy complications but is not associated with recurrent abortions.
Other options-
- Genetic abnormality- chromosomal abnormalities in either the fetus or the parents can lead to repeated miscarraiges.
- Uterine abnormality- structural issues with the uterus, such as fibroids or septate uterus, can interfere with implantation or pregnancy maintainance.
- Thyroid dysfunction- hormonal imbalances caused by thyroid problemscan lead to recurrent abortions.
Q.235. DIC occurs in all, EXCEPT?
Correct Answer : C
While uncontolled diabetes can increase the risk of certain pregnancy complications, it's not a direct cause of DIC.
The other options are more directly associated with DIC.
Q.236. Pregnancy induced HTN, all are true EXCEPT?
Correct Answer : B
Polyuria which can be defined as production of abnormally large volumes of urine, is not a typical symptom of PIH.
Q.237. Pyelonephritis in pregnancy, all true except?
Correct Answer : A
Gentamycin is an aminoglycoside antibiotic, which is not the first-line drug for treating pyelpnephritis in pregnancy. They can cause significant harm to the fetus including hearing loss and kidney damage.
First-line options to treat pyelonephritis during pregnancy includes-
- Nitrofurantoin
- Cephalexin/Amoxicillin
Q.238. Which of the following drug can be used safely during pregnancy?
Correct Answer : B
Cephalexin is a first generation cephalosporin. It is generally considered safe during pregnancy.
Q.239. Post pill amenorrhea, all true except?
Correct Answer : D
Post-pill amenorrhea is the abscence of menstrual bleeding for more than three cycles after stopping the oral contraceptive. Its more common in women who had regular periods before starting the pill.
Q.240. Regarding Infertility, all are true except?

Correct Answer : A
In Britain, male factor infertility accounts for 25% of infertile couples, while 25% remain unexplained. 50% are female causes with 25% being due to anovulation and 25% tubal problems/other.
Q.241. Vaginal trichomoniasis, all are true EXCEPT?
Correct Answer : A
While diabetes can increase the risk of various infections due to weakened immune systems, there is no strong evidence that suggest its contribution in causing vaginal trichomoniasis. The infection is primarily linked to sexual transmission.
Q.242. Primary amenorrhea occurs due to which of the following?
Correct Answer : D
Primary amenorrhea: No menses by the age of 14 and absence of secondary sexual characteristic or no menses by the age of 16 with the presence of secondary sexual characteristic.
Causes: Gonadal dysgenesis 30%, Hypothalamic-pituitary failure e.g -Kallmann syndrome (deficient GnRH), congenital absence of uterus (20%) “Agenesis of Mullerian system”, Androgen insensitivity (10%).
Q.243. In a 16-year-old pregnant lady, which of the following is least likely to be a complication of her pregnancy?
Correct Answer : D
Infant mortality is defined as death of infant before it reaches one year of age. While pregnancy can increase increase the risk of infant mortality, it is not directly a complication of pregnancy itself.
Q.244. All of the following are risk factors of puerperal infection, EXCEPT?
Correct Answer : A
Predisposing factors-
- prolonged and premature rupture of the membranes
- prolonged (more than 24 hours) or traumatic labor
- cesarean section
- frequent or unsanitary vaginal examinations or unsanitary delivery
- retained products of conception
- hemorrhage, and maternal conditions, such as anemia or debilitation from malnutrition.
Q.245. What is true regarding transdermal estrogen and OCP?
Correct Answer : A
- Tansdermal estrogen is a type of HRT where estrogen is delivered through the skin via a patch. Patches are typically replaced once or twice a week.
- OCP are birth control pills that need to be taken daily and preferably at the same time.
Patches are applied once or twice a week making them easier to remember and use.
Q.246. Indication of hepatitis in pregnancy, has high level of?
Correct Answer : C
LFT during normal pregnancy:
- Decrease total protein and albumin.
- Increase in liver-dependent clotting factors.
- Increase in transport proteins ceruloplasmin, transferrin, and globulin.
- ALP increased by 2-4 folds.
- AST/ALT should remain normal.
- Bilirubin should remain normal.
Q.247. Toxemia in pregnancy, all are true EXCEPT?
Correct Answer : C
Toxemia in pregnancy is a severe condition that sometimes occurs in the later weeks of pregnancy.
- It is characterized by high blood pressure; swelling of the hands, feet, and face, and excessive protein in the urine.
- If the condition is allowed to worsen, the mother may experience convulsions and coma, and the baby may be stillborn.
Risk factors: 1) Primigravida, 2) Previous experience of gestational hypertension or preeclampsia, 3) Family history of preeclampsia, 4) Multiple gestation, 5) women younger than 20 years and older than age 40, 6) Women who had high blood pressure or kidney disease prior to pregnancy, 7) Obese or have a BMI of 30 or greater.
Q.248. Which of the following is true regarding pre-eclampsia?
Correct Answer : C
Pre-eclampsia is a condition that can develop during pregnancy, characterized by high blood pressure and the presence of protein in the urine. It can cause a range of symptoms including headache and blurred vision.
Q.249. Infertility due to endometriosis, what is the treatment?
Correct Answer : B
Laparoscopy --> the best “surgical treatment”
Q.250. Pregnant patient with hepatitis, how is it diagnosed?
Correct Answer : D
- Screening: hepatitis surface antigen.
- After screening do liver function tests and a hepatitis panel.
Q.251. Patient with history of prolonged heavy bleeding 2 hours postpartum, what will you give?
Correct Answer : C
Prolonged heavy bleeding post partum is indicative of significant blood loss. This can lead to hypovolemic shock.
NS + PRBCis the most appropriate treatment in this scenario as-
- NS restores the blood volume and maintain blood pressure.
- PRBC replace the lost red blood cells which are responsible for carrying oxygen throughout the body.
Q.252. A 25-year-old pregnant presented with fever and sore throat (in flu season), then she developed non productive cough and dyspnea, she was extremely hypoxic. What is the most likely diagnosis?
Correct Answer : B
Flu-like symptoms progressing to severe respiratory distress, non-productive cough, and hypoxia suggest staphylococcal pneumonia, especially in high- risk individuals like pregnant women.
Q.253. A patient presented with PV bleeding, how will you differentiate between abruptio placenta and spontaneous abortion?
Correct Answer : C
Abruptio placenta typically causes sudden gush of blood with severe pain due to placental sepration, while spontaneous abortion usually leads to lighter, gradual bleeding.
Q.254. A 14-year-old girl complains of painless vaginal bleeding for 2-4 days every 3 weeks for 2 months ranging from spotting to 2 pads per day, she had developed secondary sexual characteristic 1 year ago and had her menstruation since 6 months. On clinical examination she has normal sexual characteristics, normal pelvic exam. What is your appropriate action?
Correct Answer : A
The onset of mensturation can be irregular in the first few years after it begins. This is due to hormonal fluctuations as the body establishes a regular ovulatory cycle.
OCPs can be used for-
- Regulating the menstrual cycle.
- Reducing the amount of duration and bleeding.
Q.255. A women with history of multiple intercourse has ulcer on cervix. What is the 1st line investigation?
Correct Answer : A
Pap smear is a simple test that checks for abnormal cells on cervix. It's the first-line investigation for any changes in cervix, including ulcers.
Q.256. A 14-year-old presented with irregular bleeding per vagina, she started to have secondary sexual characters 9 months ago, and menarche 6 months ago. Her menstruation differs from spotting to two pads per day. She denies sexual activity, stress, and severe exercise. What will you do?
Correct Answer : A
Irregular bleeding after menarche for the first few years is common. this is because her body is still establishing a regular ovulation cycle.
Q.257. Surveillance of patient on hormone replacement therapy includes all of the following, EXCEPT?
Correct Answer : C
Surveillance of patient on HRT includes the following-
- BP monitoring [can sometimes raise BP]
- Breast examination [as increases the risk of breast cancer]
- Pelvic examination
Q.258. Age of menopause is predominantly determined by which of the following?
Correct Answer : D
Menopause is the natural cessation of menstruation, marking the end of a woman's reproductive years. The age at which menopause primarily occurs is determined by genetics.
Q.259. A pregnant lady presents with no fetal movements; her CBC shows platelets- 75000. What is the diagnosis?
Correct Answer : A
Low platelet levels--> marker for pre-eclampsia, autoimmune diseases such as systemic lupus erythematosus (SLE) and Idiopathic Thrombocytopenic Purpura (ITP).
Elevated platelet levels--> may indicate thrombocythemia.
Q.260. Following evacuation of a molar pregnancy, B-hCG titers will fall to undetectable levels in about 90% of patient within how many weeks?
Correct Answer : D
Molar pregnancy is a rare condotion in which the placenta develops abnormally. Following the removal of molar pregnancy by suction and evacuation the level of hCG should gradually decrease.
In 80% cases the levels will fall to undetectable levels within 12-16 weeks after evacuation.
Q.261. A 16-year-old female presents to your clinic with chief complaints of never having had a menstrual period. She never had done a pelvic examination. Physical exam reveals the following: BP-110/70, pulse- 72, weight- 60kg & Ht-172. The patient appears her stated age. Axillary and pubic hair is scanty. Breasts are tanner stage IV. External genitalia are normal female. A mass is palpable within the inguinal canal. Pelvic exam reveals an absent cervix with the vagina ending in a blind pouch. The uterus and ovaries are difficult to delineate. What is the likely diagnosis?
Correct Answer : D
AIS occurs when the body cannot respond to male hormones ,leading to female external genitalia despite XY chromosomes. It causes primary amennorhea because functional ovaries do not develop.
Incorrect options-
- Prolactin secreting aadenoma- amenorrhea + galactorrhea + infertility
- PCOS- irregular periods/amenorrhea + acne,hirsuitism, or obesity
- Turner syndrome- short stature, webbed neck, ovarian faliure etc.
Q.262. To confirm the diagnosis of AIS, which test would be most readily obtained by ordering?
Correct Answer : D
In AIS, the karyotype would typically show XY chromosomes, confirming the genetic basis of condition.
Q.263. A lady presents with post coital spotting and dysuria. What is the organism that likely causes this condition?
Correct Answer : A
Chlamydia trachomatis is a common STI that can cause post coital spotting and dysuria.
Q.264. Karyotype is performed on a male patient's peripheral blood lymphocytes. What is the karyotype?
Correct Answer : C
46,XY-male karyotype
Incorrect options-
- 46,XX- female
- 45,X- Turner syndrome
Q.265. A patient presents with erythematous cervix with punctate areas of exudation (strawberry cervix ). What is the causative organism?
Correct Answer : A
Trichomonas vaginitis presents with the typical feature of strawberry cervix.
Q.266. The hormone profile in a Klinefelter patient would include all of the following, EXCEPT?
Correct Answer : D
Hormone profile of a female with Klienfelter syndrome include-
- Elevated LH as the testes are not producing enough testosterone for a negative feedback to the pituitary gland.
- Normal to elevated FSH due to impaired testicular function.
- Elevated estradiol
Q.267. An inguinal mass most likely represents which of the following?
Correct Answer : C
The mass in the groin is most likely an undescended testes [cryptorchidism]. Undescended testes often show abnormal development, including an increase in Leydig cells and an abscence of spermatogenesis.
Q.268. The most long term treatment would be?
Correct Answer : B
It is the most long-term treatment option because it addresses the underlying hormonal imbalance. This therapy helps in replacing estrogen and hence alleviates the symptoms.
Q.269. All of the following are true about a patient with AIS [46,XY] except?
Correct Answer : D
30% of women with a Y chromosome do not have virilization. Androgen insensitivity (10%), which is also known as MALE pseudohermaphroditism: the genitalia are opposite of the gonads. Breasts are present but a uterus is absent. Such individuals have 46, XY karyotypes with a body (incomplete forms) that lacks androgen receptors.
Mullerian inhibitory factor, produced by the testis results in involution of the 5th mullerian duct and its derivatives. So there will be an external genitalia development, axillary and pubic hair growth is dependent on androgen stimulation. Because no androgen is recognized by the body, there will be no pubic &axillary hair development. Female breast develops in response to the estrogen normally produced by male testes.
Examination: normal female phenotype, but no pubic or axillary hair growth, Short blind vaginal pouch, no uterus, cervix, or proximal vagina. Undescended testes are palpable in the inguinal canal. Diagnosis is confirmed by normal male testosterone levels and a normal male 46,xy karyotype.
Management: is by neovagina, gonads should be removed and estrogen replacement therapy should be then administered.
Q.270. Without surgery a patient with mixed gonadal dysgenesis [MGD] is at a risk to develop which of the following?
Correct Answer : B
Gonadoblastoma are tumors that can arise in gonads of people with MGD. They are considered precancerous , but they are not the immediate risk without surgery and have the potential to develop into more agressive cancers, such as dysgerminoma.
Q.271. All of the following result from combined estrogen-progestin replacement therapy, EXCEPT?
Correct Answer : D
Recent controlled, randomized study found HRT may actually prevent the development of heart disease and reduce the incidence of heart attack in women between 50 and 59, but not for older women.
Q.272. All of the following are characteristic changes seen in menopause, EXCEPT?
Correct Answer : A
Changes in menopause include:
- Changes in menstrual cycle (longer or shorter periods, heavier or lighter periods, or missed periods)
- Hot flashes, Night sweats (hot flashes that happen while sleep)
- Vaginal dryness
- Sleep problems
- Mood changes (mood swings, depression, irritability)
- Pain during sex, less interest in sex
- More urinary infections, Urinary incontinence
- Increase in body fat around the waist
- Problems with concentration and memory.
Q.273. Management of possible ruptured ectopic pregnancy would include all of the following, EXCEPT?
Correct Answer : D
Observation and methotrxate are used for unruptured ectopic pregnancy. Ruptured ectopic pregnacy is a medical emergency and should immediately be managed by appropriate resuscitative measures and immediate sugical intervention.
Q.274. A 25-year-old G3P1 presents to the emergency room complaining of lower abdominal pain, her LMP was 6 weeks back. She has had significant vaginal bleeding but no passage of tissue. BP - 100/70. What is the most likely diagnosis?
Correct Answer : D
All the above mentioned symptoms are consistent with the diagnosis of ectopic pregnancy, where the fertilized egg implats outside the uterus, typically in the fallopian tube.
Q.275. For pregnancy induced hypertension, all are true except?
Correct Answer : A
Birth control pills do not increase the risk of PIH. Infact, they can sometimes be used to manage the condition.
Q.276. What is the most important step in a pregnant patient's evaluation?
Correct Answer : A
Evaluating a patient with pregnacy- related concerns, sonography is crucial. It provides information regarding location and viability of the pregnancy. It can help determine if the pregnancy is intrauterine or ectopic, . This is essential for guiding further management and treatment.
Q.277. How do you accurately diagnose hydatidiform mole?
Correct Answer : B
Diagnosis is based on a typical sonographic “snowstorm” pattern.
The following findings also support a diagnosis of hydatidiform mole:
- Absence of a gestational sac, by ultrasound assessment, or absence of fetal heart tones, by Doppler, after 12 weeks.
- Pregnancy test showing elevated human chorionic gonadotropin (hCG) serum levels greater than 100,000 IU.
- Development of preeclampsia prior to 20 weeks.
- Uterine size greater than estimated gestational size.
- Vaginal bleeding.
Q.278. After the B-hCG titre becomes undetectable, the patient treated for hydatidiform mole should be followed up for how long?
Correct Answer : C
hCG levels should be monitored for 1 year, This helps ensure the mole has resolved completely and allows early detection of persistent disease or choriocarcinoma, which can arise if the hCG levels fail to normalize or start increasing again.
Q.279. What would the transvaginal ultrasonography in case of missed abortion likely reveal?
Correct Answer : D
Missed abortion is a condition in which the fetus has stopped developing but the products of conception remain in the uterus. A transvaginal scan would likely show a thickened endometrium but no visible gestational sac, because the pregnancy has failed to progress and the gestational sac may be indistinguishable.
Q.280. Ectopic pregnancy can be ruled out with a high degree of certainty in presence of which of the following?
Correct Answer : D
An ectopic pregnancy occurs when the fertilized egg implants outside the uterus, typically in the fallopian tube. If a transviginal ultrasound shows a geastational sac within the uterus it is strongly suggestive of ruiling out ectopic pregnancy.
Q.281. An Rh -ve woman married to an Rh +ve man should receive Rh immunoglobulin under which of the following conditions?
Correct Answer : C
Ectopic pregnancy- fetal blood mixing can occur risking Rh sensitization. Rh immunoglobulin prevents this.
ECV- this procedure can cause fetal-maternal blood mixing, so Rh immunoglobulin is needed as a precaution.
Q.282. Physical examination reveals 6 weeks size uterus. Vaginal bleeding is scanty with no discernible tissue in the cervical os. There are no palpable adnexal masses. The uterus is mildly tender. Ultrasonographic exam does not reveal a gestational sac. Which of the following should be recommended?
Correct Answer : D
Diagnostic laproscopy allows direct visualization of the pelvic organs.
Q.283. Vomiting in pregnancy, all of the following are true, EXCEPT?
Correct Answer : A
In most of the cases vomiting can be mild and managed at home with lifestyle changes and OTC drugs.
Q.284. What is the most common presenting symptom of ectopic pregnancy?
Correct Answer : B
The classic symptoms of ectopic pregnancy are:
1) Abdominal pain, 2) Amenorrhea, 3) Vaginal bleeding.
These symptoms can occur in both ruptured and un-ruptured cases.
In one representative series of 147 patients with ectopic pregnancy (78 % were ruptured), abdominal pain was a presenting symptom in 99 %, amenorrhea in 74% and vaginal bleeding in 56%.
Q.285. If the above patient presented at 8 weeks gestation & pelvic exam revealed unilateral adnexal tenderness w/o discernible mass. What should be done?
Correct Answer : C
Laparoscopy is the gold standard for diagnosing and treating ectopic pregnancy.
Q.286. What is the common location of ectopic pregnancies?
Correct Answer : A
Fallopian tube- Ampullary region[70-80%], isthmic reigon[12%], fimbrial reigon[5%], interstitial reigon[2-4%].
Non-tubal sites- ovarian[3%], abdominal[1%], cervical[<1%], C-section scar[<1%].
Q.287. Drugs that should be avoided during pregnancy includes all of the following, EXCEPT?
Correct Answer : B
Cephalexine is a first-generation cephalosporin antibiotic. It is generally considered safe to use during pregnancy, especially in the second and third trimester.
Q.288. If a woman with preeclampsia is not treated prophylactically to prevent eclampsia, her risk of developing seizure will be?
Correct Answer : D
If a woman with preeclampsia is not treated prophylatically to prevent eclampsia her risk of developing a seizure is approximately 1 in 200.
Q.289. If the above described patient has had a previous term pregnancy prior to her current ectopic pregnancy, her chances of subsequent intrauterine pregnancy would be about ?
Correct Answer : A
Those with previous normal pregnancy have about 80% chance after their ectopic pregnancy to achieve intrauterine pregnancy.
- A study of surgical and medical therapy of ectopic pregnancy reported the rates of recurrent ectopic pregnancy after single-dose methotrexate, salpingectomy, and linear salpingostomy were 8, 9.8, and 15.4 percent, respectively.
- Women who have had conservative treatment for ectopic pregnancy are at high risk (15 % overall) for recurrence.
Q.290. In normal pregnancy, the value of B-HCG doubles every?
Correct Answer : A
Studies in viable intrauterine pregnancies have reported the following changes in serum hCG:
- The mean doubling time for the hormone ranges from 1.4 to 2.1 days in early pregnancy.
- In 85 percent of viable intrauterine pregnancies, the hCG concentration rises by at least 66 percent every 48 hours during the first 40 days of pregnancy.
Q.291. HELLP syndrome is characterized by all of the following except?
Correct Answer : D
Features of HELLP syndrome-
- Thrombocytopenia (<100,000) due to hemolysis
- Elevated liver enzyme levels
- Low platelet count (<150)
Q.292. All of the following are seen in utero with alloimmune hydrops, EXCEPT?
Correct Answer : D
This is a common complication of alloimmune hydrops, but it is not seen in utero. Bilirubin levels usually start to rise after birth.
Q.293. A serum progesterone value <5 ng/ml can exclude the diagnosis of a viable pregnancy by?
Correct Answer : D
A meta-analysis of 26 studies on the performance of a single serum progesterone measurement in the diagnosis of ectopic pregnancy found that a level less than 5 ng/mL (15.9 nmol/L) was highly unlikely to be associated with a viable pregnanc.
[Only 5 of 1615 patients (0.3 percent) with a viable intrauterine pregnancy had serum progesterone below this value].
Q.294. What is the most common presenting prodromal sign or symptom in patients with eclampsia?
Correct Answer : C
Headache is a very common symptom that precedes the more serious symptoms of eclampsia such as seizures, and coma.
Q.295. Which of the following drugs does not cross the placenta?
Correct Answer : A
Heparin is a large molecule and does not cross placenta, and therefore is safe to use during pregnancy.
Common drugs with their side effects-
- Chloramphenicol causes- ''Gray baby syndrome''
- Tetracycline- teeth defects in the child
- Warfarin- birth defects
- Diazepam- exaggerated reflexes in the newborn
- Aspirin- intracranial bleeding
Q.296. What is the most consistent finding in a patient with eclampsia?
Correct Answer : D
Features of eclampsia include:
- Seizure or postictal status [100%]
- Headache [80%]
- Generalized edema [50%]
- Vision disturbance [40 %]
- Abdominal pain with nausea [20%]
- Amnesia & other mental status changes
Q.297. Risk factors for HSV2 in infants includes all of the following, EXCEPT?
Correct Answer : C
Maternal bodies for HSV-1 do not provide protection against HSV-2 as both of them are different viruses.
Q.298. Eclampsia occurring before 20 weeks of gestation is most commonly seen in which condition?
Correct Answer : C
Eclampsia prior to 20 weeks gestation is rare & should raise the possibility of underlying molar pregnancy or antiphospholipid syndrome.
Q.299. Appropriate responses to an initial eclamptic seizure includes all of the following, EXCEPT?
Correct Answer : A
The goal of management is to limit maternal and fetal morbidity until delivery of the neonate, the only definitive treatment for eclampsia.
Supportive care for eclampsia consists of -
- Close monitoring, invasive if clinically indicated, airway support, adequate oxygenation, anticonvulsant therapy, and BP control.
- Magnesium sulfate is the initial drug administered to terminate seizures.
Compared with the traditional drugs used to terminate seizures (e.g., diazepam, phenytoin [Dilantin]), magnesium sulfate has a lower risk of recurrent seizures with non-significant lowering of perinatal morbidity and mortality.
Q.300. What is the most common cause of infection?
Correct Answer : B
A retained placenta is a condition in which a part or whole of the placenta remains in the uterus after childbirth. This can create a breeding ground for bacteria, leadin to infection.
Complications-
- Primary complications- PPH, uterine rupture, infections [endometritis,sepsis].
- Long term complications- Infertility, Ashermann's syndrome.
Q.301. Which one of the following drugs is safe in pregnancy?
Correct Answer : C
Erythromycin is generally considered safe during pregnancy because extensive use has not shown an incresase in fetal malformations or harm. It is often prescribed to pregnant ladies who are allergic to penicillin.
Nitrofurantoin not safe in term period, labour or during delivery- reason : it can cause haemolysis in fetus
Q.302. The likely contributory mechanisms of the anticonvulsant action of MgSO4 include all of the following, except?
Correct Answer : B
Peripheral neuromuscular blockade is not a direct contributory mechanism of it's anticonvulsant effects. While MgSO4 can relax muscles at higher doses, that is unrelated to its seizure control properties.
Q.303. Which of the following is true about gonoccocal infection?
Correct Answer : B
Gonococcal infection can spread to the fallopian tubes in women, causing inflammation and scarring. This scarring can lead to tubal blockage.
Q.304. Non-contraceptive use of combined oral contraception is used in which of the following?
Correct Answer : D
Combined oral contraceptives have a variety of non-contaceptive uses, including the treatment of menorrhagia, paimary dysmenorrhea, and fuctional small ovarian cysts.
Q.305. All of the following antihypertensive medications are considered safe for short term use in pregnancy, except?
Correct Answer : A
Complications seen with fetuses exposed to captopril:
1) Low blood pressure (hypotension), 2) Developmental problems with the nervous system, 3) Developmental problems with the cardiovascular system, 4) Developmental problems with the lungs, 5) Kidney failure, 6) Deformities of the head and face, 7) Loss of life.
These drugs should be avoided during pregnancy:
1) Alcohol, 2) Antianxiety agents (fluoxetine is now the drug of choice for anxiety and depression during pregnancy), 3) Anti-neoplastic agents, 4) Anticoagulants (coumarin derivative like warfarin) but heparin can be used because it does not cross placenta, 5) Anticonvulsants “Carbamazepine and valproic acid are associated with increased risk for spina bifida” , 6) Diuretics, 7) Retinoid.
Q.306. The reason to treat severe chronic hypertension in pregnancy is to decrease?
Correct Answer : D
- Risks of severe chronic hypertension in pregnancy is that it affects the mother more. It may include, but is not limited to, the following:
1) blood pressure increasing, 2) congestive heart failure, 3) bleeding in the brain, 4) kidney failure, 5) placental abruption (early detachment of the placenta from the uterus), 6) blood clotting disorder.
- Risks to the fetus and newborn depend on the severity of the disease and may include, but are not limited to, the following:
1) Intrauterine growth restriction (IUGR) - decreased fetal growth due to poor placental blood flow, 2) pre-term birth (before 37 weeks of pregnancy), 3) stillbirth.
Q.307. An Rh- ABO incompatible mother delivers an Rh+ infant at term and does not receive Rh immune globulin. The probability of detection of anti-D antibody during her next pregnancy is about?
Correct Answer : D
Isoimmunization occur when:
- Rh negative women pregnancy with Rh+ baby
- Sensitization rants
- Incompatible blood transfusion
- Fetal placental hemorrhage (ectopic pregnancy)
- Any type of abortion
- Labor and delivery
Isoimmunization occuring for 1st child, risk for the next pregnancy is 16% which is reduced by exogenous Rh IgG given to the mother to less than 2% Anti-Rh IgG crosses the placenta and can cause fetal RBC hemolysis which causes (anemia – CHF – edema – ascites) and in severe case cause, fetal hydrops or erythroblastosis fetalis.
Q.308. What is the possible mechanism of action of intrauterine contraceptive devices?
Correct Answer : D
Mechanism of action -
1) Copper produce alterations of the uterine environment in terms of a pronounced foreign body reaction. Disrupting sperm mobility and damaging sperm, 2) progesterone reduce menstrual bleeding or prevent menstruation, thickened cervical mucus may suppress ovulation.
Absolute contraindication: 1) Pregnancy, 2) undiagnosed vaginal bleeding, 3) acute or chronic pelvic inflammatory disease, 4) risk of STDs, 5) Immunosuppressant, 6) willsons disease and allergy to copper
Relative contraindication: 1) Valvular heart disease, 2) Past medical history of ectopic pregnancy or PID, 3) Presence of prosthesis, 4) Abnormality of uterus cavity, 5) Sever dysmenorrhea or menorrhea, 6) Cervical stenosis.
Q.309. What is true about Vulvovaginal candidiasis?
Correct Answer : C
Vulvovaginal Candidiasis: vulvar pruritis or vulvar burring with abnormal vaginal discharge “thick curd-like”. It is common in pregnant women. It is a local infection (No systemic infection).
Diagnosis: by microscopic examination with KOH, Culture, and Pap smear Vaginal PH < 4.5.
Treatment: by 1st line antifungal oral fluconazole 2nd line * anti-fungal oral nystatin * Boric acid (locally). (a sole drug contraindicated in pregnancy) It is not a sexually transmitted infection it is associated with it.
Q.310. What is correct about Bacterial vaginosis?
Correct Answer : D
Bacterial vaginosis: shift from a healthy lactobacilli-based endogenous flora to anaerobically based endogenous flora (the rectum is the source of infection). It presents as gray white vaginal discharge,and fishy odor.
Diagnosis- by Vaginal PH > 4.5, affirm VP microbial identification, Cytology, Absence of lactobacilli in gram stain.
Treatment by metronidazole & Clindamycin.
Q.311. What is true about Chlamydia trachomatis infection?
Correct Answer : D
Chlamydia: Infection by chlamydia trachomatis. It is a STD.
Symptoms: mucopurulent vaginal discharge, urethral symptoms “dysuria, pelvic pain, pyuria, frequency, post coital bleeding and conjunctivitis in infants.
Diagnosis-culture, PCR, Direct immature antibody test.
Treatment - doxycyclin/tetracycline/azithromycin.
STDs: 1) chlamydia 2) gonorrhea 3) genital warts 4) syphilis 5) Herpes simplex of vulva.
Q.312. For progesterone only contraceptive pills, which of the following is correct?
Correct Answer : D
Progesterone OCP is used in -
1) Postpartum [Breast feeding]
2) Women with myocardial disease
3) Women with thromboembolic disease
4) Women who can not tolerate combined OCP (estrogen side effect)
Q.313. The prevalence of gestational diabetes in the general population is about?
Correct Answer : B
Prevalence of 4% is more common is.
Q.314. What is the most common cause of polyhydramnios?
Correct Answer : D
Polyhydramnios- amniotic volume >2000cc at any stage.
- Causes-
1) Idiopathic [most common], 2) Type 1 DM, 3) Multiple gestation, 4) Fetal hydrops, 5) Chromosomal anomaly, 6) Malformed lung, 7) Duodenal atresia
- Complication-
1) Cord prolapse, 2) Placental abruption, 3) Malpresention, 4) Preterm labor, 5) Postpartum hemorrhage.
- Diagnosis - aminocentasis
- Treatment [if it is severe]--> amniocentesis [ but if mild to moderate no treatment].
Q.315. Gestational diabetes is associated with which of the following?
Correct Answer : D
Gestational diabetes can lead to excessive glucose in the baby's blood stream. This can cause the baby to grow too large in size leading to a condition called macrosomia. In macrosomia the baby's head and abdomen may grow disproportionately compared to the rest of their body. This can result in a decreased head cirumference and abdominal circumference ratio.
Other complications in the option are not typically associated with macrosomia.
Q.316. Generally accepted cutoff values for plasma glucose on the 1000 gm, 3-hour glucose tolerance test in pregnancy (according to the National Diabetes Group) include all of the following, EXCEPT?

Correct Answer : A
The NDDG guidelines for gestational diabetes diagnosis use a 3-hour OGTT. Option A is an outlier as a fasting glucose level above 90 mg/dl alone does not meet the diagnostic threshold.
Q.317. Normal pregnancy in the 2nd trimester is characterized by all of the following, EXCEPT?
Correct Answer : B
During second trimester, a woman's body undergoes many changes to support the growing baby. One of these changes is an increase in insulin senstivity. As a result fasting plasma glucose levels tend to be lower in the second trimester compared to before pregnancy.
Q.318. Infants of mothers with gestational diabetes have an increased risk of all of the following, EXCEPT?
Correct Answer : B
GDM is when a women develops high blood glucose levels during pregnancy. When a baby is exposed to high levels of glucose in womb, their pancreas start producing more insulin to manage it. After birth, the baby's insulin levels can remain high leading to hypoglycemia, not hyperglycemia.
Q.319. Gestational diabetes is associated with an increased risk of all of the following, EXCECPT?
Correct Answer : D
GDM is associated with an increased risk of macrosomia [baby is larger than average size], not smaller. IUGR is when the baby doesn't grow as expected in the womb, and it's not typically linked to gestational diabetes.
Q.320. Infants of mothers with gestational diabetes are at increased risk of becoming which of the following?
Correct Answer : C
The current research on the long-term effects of gestaional diabetes on the children is inconclusive regarding the risk of obesity and type 2 diabetes.
Q.321. A post menopausal women presents with itchy, pale, and scaly labia minora. What is the diagnosis?
Correct Answer : A
It is a skin condition caused by repeated scratching or rubbing of the skin. This constant irritation leads to thickening, scaling, and inflammation of the skin. In this case, the itching of the labia minora likely led to the woman scratching the area, causing the symptoms described.
Q.322. Control of gestational diabetes is accomplished with all of the following, EXCEPT?
Correct Answer : C
Oral hypoglycemic agents are contraindicated in pregnancy.
Q.323. Type I diabetes is associated with all of the following, EXCEPT?
Correct Answer : D
Women with type 1 diabetes have a higher risk of IUGR. This is because poor blood sugar control can lead to insuffficient nutrient delivery to the fetus, hindering its growth.
Q.324. What is classical characteristic of genital herpes?
Correct Answer : A
This is the hallmark sign of genital herpes. The virus causes small painful blisters [vesicles] that often break open to form ulcers.
Q.325. Which of the following is true regarding infertility ?
Correct Answer : C
Prolactin is a hormone that plays a role in milk production. High levels of prolactin can interfere with ovulation in woman, making it harder to concieve. This is known as hyperprolactinemia and can be a cause of infertility.
Q.326. A 32-year-old female patient presented with irregular menses, her menses occurs every two months. On examination everything is normal, which of the following is the ''LEAST'' important test?
Correct Answer : C
Irregular menstrual cycles, while a concern, are not typically associated with bleeding disorders. A coagulation profile investigates blood clotting factors, which wouldn't be the first line of testing for the patient's specific presentation.
Q.327. Which of the following is true for a pregnant women?
Correct Answer : C
Reflux esophagitis, or acid reflux can lead to iron deficiency anemia in pregnant women. This is because the acid in the stomach can irritate the lining of the esophagus causing bleeding and iron loss.
Q.328. A 19-year-old female presents with depression, anxiety, and mood swings affecting her life. She experiences these symptoms every month before menstruation. What is the most appropriate treatment?
Correct Answer : A
SSRIs are a class of antidepressants commonly used in the treatment of PMDD [premenstrual dysmorphic disorder], a severe form of premenstrual syndrome characterized by depression, anxiety, and mood swings. A SSRI helps regulate serotonin levels, which is a neurotransmitter that plays a role in mood regulation.
Q.329. A pregnant lady develops HTN, drug of choice for HTN in pregnancy is?
Correct Answer : A
Methyl-dopa is considered the first-line drug for treating HTN in pregnancy.
Q.330. A 35-year-old primigravida is at 16 weeks of gestation. She is coming for her 1st check up, she is excited about her pregnancy. No history of any previous disease. Her BP at rest is 160/100mmHg, after one week her BP is 154/96mm/Hg. What is the most likely diagnosis?
Correct Answer : A
The patient is 35 years old and has a history of no previous disease. Her BP at rest is elevated at 160/100 mmHg. This suggests that she had a high BP before becoming pregnant. Since the HTN existed before pregnancy, it's classified as chronic HTN.
Q.331. A 9 month pregnant lady presents with full cervical dilation and head of the baby is seen, there is late fetal heart deceleration seen on CTG. What will be the next step in management?
Correct Answer : B
The precense of late fetal heart decelartion on CTG is indicative of fetal distress. As the baby's head is already low in the birth canal forceps can be used to perform a controlled delivery in order to minimize further stress on the baby and expediate the process.
Q.332. A patient presents with idiopathic anovulation. Which of the following drug will you give to the patient?
Correct Answer : A
Clomiphene citrate is the first-line drug used to treat anovulation. It works by blocking the estrogen receptors in the hypothalamus, which in turn tricks the body into thinking that it has low estrogen levels. This triggers the release of more FSH from the pituitary gland, stimulating ovulation.
Q.333. A patient presents with symptoms consistent with Haemophilus ducreyi infection. What is the recommended treatment approach for their sexual partners?
Correct Answer : A
Haemophilus ducreyi is a STI that causes painful genital ulcers. It's crucial to treat all sexual partners, even if they are asymptomatic, to prevent the spread of infection and the development of complications like scarring and infertility.
Treatment-
- Azithromycin- 1g in a single dose.
- Ceftriaxone- 250mg I/M in a single dose.
- Ciprofloxacin- 500mg orally twicw a day for 3 days.
- Erythromycin- 500mg 3 times a day for 7 days.
Q.334. What is true about dysfunctional uterine bleeding?
Correct Answer : B
DUB can occur in adolscent girls. During puberty, their hormonal system is still establishing a regular pattern, and irregular bleeding is common. This can manifest as heavy bleeding, prolonged bleeding, or bleeding between periods.
Q.335. Which of the following contraception affects lactation?
Correct Answer : A
Combined contraceptive pills contain both estrogen and progesterone, which tends to affect lactation in several ways-
- estrogen can decrease milk production and change milk composition.
- estrogen can also cause delayed lactogenesis.
Q.336. Which of the following is a cost effective & temporary contraceptive method?
Correct Answer : B
- Cost-effective- Condoms are generally inexpensive and readily available.
- Temporary- They provide immediate protection .
Q.337. Which of the following is the most common cause of vaginal bleeding?
Correct Answer : A
Menstraution- It is defined as the normal shedding of the uterine lining [endometrium] each month in every women of reproductive age who are not pregnant. It's a natural and regular occurence.
Q.338. A female patient with irregular menstrual cycle, that occurs every other month and lasts for 7-8 days with very heavy bleeding requiring her to use double pads, yet these pads get soaked completely. What is the term used for this condition?
Correct Answer : D
Metromenorrhagia- It refers to the occurence of both metorrhagia [intermenstrual bleeding], and menorrhagia [excessive menstrual bleeding] at the same time.
Q.339. Which of the following has association with PCOS?
Correct Answer : A
PCOS is strongly associated with androgen excess, with male sex hormones such as testosterone. This excess can lead to symptoms like hirsuitsm, acne, and irregular periods.
Q.340. A young lady presented with vaginal bleeding, her vaginal examination showed a white cervical lesion. A diagnosis of cervical cancer was made. What is the next step?
Correct Answer : D
In this procedure the doctor uses a special magnifying instrument with a light source to get closer look at the cervix and obtain a biosy if needed. It's the next step in investigating the suspicious leison to confirm the diagnosis and determine the extent of cancer.
Q.341. Which of the following is the first-line drug for the management of PPH?
Correct Answer : A
Uterine contactiles like oxytocin are the first-line medication for PPH. They work by causing the uterus to contract, leading to compression of blood vessels and thereby cause the bleeding to stop.
Q.342. During the third trimester of pregnancy, all of the following changes occur normally, except?
Correct Answer : B
Pregnancy in the final month, may be associated with increased WBC levels.
Q.343. A 40-year-old female, nulligravida, has no history of sexual intercourse for 1 year because her husband lives abroad. She C/O intermenstrual bleeding with menorrhagia and severe dysmenorrhea. What is the most likely diagnosis?
Correct Answer : C
This condition involves growth of endometrial tissue in the myometrium. This can cause heavy bleeding, painful periods, and intermenstrual bleeding.
Incorrect options-
- Endometriosis- The patient's lack of sexual intercourse for a year makes it less likely.
- Endomentrial cancer- More common in postmenopausal women.
- Fibroids- Less likely to cause intermenstrual bleeding.
Q.344. A 34-year-old female at 34 weeks of GA, came with vaginal bleeding. Previous pregnancies were normal with normal deliveries. She is a known smoker. What will you suggest?
Correct Answer : A
Smoking during pregnancy is a major risk factor for many complications, including preterm labor, and placental problems, which can lead to vaginal bleeding. Stopping smoking is a crucial step to reduce these risks and improve the overall health of the mother and baby.
Q.345. Propylthiouracil drug is contraindicated in which of the following?
Correct Answer : B
The best choice among the following is maternal DM as managing maternal diabetes is already complex and PTU's potential liver side effects could add to complications.
Q.346. A pregnant female who is a known smoker presents with h/o trauma & dark red vaginal bleeding. FHR is 150b/min. What is your diagnosis?
Correct Answer : B
The dark red vaginal bleeding and history of trauma are strong indicators of abruptio placenta.
Q.347. What's the most common cause of secondary amenorrhea, FSH >> LH?
Correct Answer : A
During menopause the ovaries stop producing estrogen and progesterone. This leads to a decline in ovarian function which in turn leads to elevated levels of FSH compared to LH. This is a characterstick hormonal ratio pattern in menopause.
Q.348. A female presents with vaginal bleeding, and abdominal pain. What's the first line investigation?
Correct Answer : A
USG is the first-line investigation in case of vaginal bleeding and abdominal pain. It provides a non-invasive way to visualize the pelvic organs and know the potential cause of symptoms.
Q.349. What is the best test to detect age of gestation?
Correct Answer : A
USG- It is the most accurate method to determine the gestational age in early pregnancy. It allows visualization of developing fetus and can measure key parameters like crown-rump length, whcih helps estimate the gestational age with high precision.
Q.350. What is the significance of the physiologic hypervolemic state of pregnancy in the management of severely injured gravid women?
Correct Answer : D
The physiological hypervolemic state means pregnant women have a larger blood volume compared to non pregnant individuals. Pregnant women can loose greater volume of blood before showing signs of hypotension. This can be misleading as initial sign of blood loss might be less apparent due to the increaded blood volume.
Q.351. Pregnant lady presented with bleeding from gums. On examination, spleen is palpable 4 cm below the costal margin. Investigations- platelets 50,000. What is the diagnosis?
Correct Answer : A
HELLP syndrome is characterized by-
- Hemolysis
- Elevated liver enzymed
- Low platelet count
- The precense of bleeding gums and an enlarged spleen further supports the diagnosis .
Q.352. A patient presents with vaginal discharge, suprapubic pain for 3 days, fever and bilateral adnexal tenderness. What is the diagnosis?
Correct Answer : B
Acute salphingitis is the inflammation of fallopian tubes, and its a complication of PID.
Q.353. A female patient with nipple discharge bilaterally is found to have pituitary mass. What is the cause?
Correct Answer : A
Prolactin is a hormone primarily responsible for stimulating milk production. Precense of pituitary mass can disrupt normal production of the hormone. High prolactin levels can cause galactorrhea in both men and women regardless of pregnancy or lactation.
Q.354. What is the most concerning symptom during pregnancy?
Correct Answer : A
Vaginal bleeding is the most concerning symptom during pregnancy.
Q.355. A patient has postpartum haemorrhage, started on ergotamine. Which of the following condition is contraindication for this medication?
Correct Answer : A
Maternal HTN can be aggrevated by ergotamine.
Q.356. What is the contraindication of Methyl-progesterone used for PPH?
Correct Answer : B
It has the tendency of slightly increasing the BP. Therefore it is contraindicated in pregnant women with pre-existing HTN, as it could further elevate the BP and worsen their condition.
Q.357. 4 days post C-section a patient was found to have profound hypotension. What is the most appropriate management?
Correct Answer : A
The immediate priority in this scenario is to restore fluid volume and improve blood pressure. Normal saline is the fluid of choice for volume resuscitation. Two big I/V lines allow rapid fluid administration, which is crucial in this situation.
Q.358. A 23-year-old female with regular menses. On USG, she has a 7 cm ovarian cyst. Otherwise, everything is normal. What is the diagnosis?
Correct Answer : B
Follicular cysts are the most common type of ovarian cyst and typically occur during a womens rproductive years. They form when a follicle fails to rupture and release the egg, causing it to grow into a cyst. Since the patient has regular menses her ovarian function is normal making a follicular cyst the most likely diagnosis.
Q.359. An obese female has missed two menstrual cycles and feels nauseated and has vomited several times , what is your next step ?
Correct Answer : A
Given the patients symptoms the most appropriate next step is to perform a blood test to confirm the diagnosis of pregnancy.
Q.360. A divorcee with a h/o amenorrhea since 15 months was found to have high FSH. What is the diagnosis?
Correct Answer : A
Primary ovarian faliure is a condition in which the ovary stops functioning normally before the age of 40. This leads to decreased estrogen production and elevated levels of FSH.
- Symptoms- amennorhea, hot flashes, vaginal dryness.
Q.361. A female presented with foul odor & greenish vaginal discharge with flagella under the microscope. What is the causative organism?
Correct Answer : A
All these features are typically seen in Trichomoniasis.
Q.362. A postmenopausal women presents with history of itching and bloody discharge, she used suppository but had no relief. She then developed a swelling on her right labia majora. What is the diagnosis?
Correct Answer : B
The combination of itching, bloody discharge, lack of relief with a suppository, and the development of a swelling on the right labia majora strongly suggests the diagnosis to be Bartholins abscess.
Incorrect options-
- Bartholin's cyst- cyst is a fluid-filled sac.
- Vulvar carcinoma- typically presents with persistent itching, burning, or bleeding. But there is no localized swelling due to which we select Bartholi's abscess as the answer.
- Endometrial carcinoma- typically presents with abnormal vaginal bleeding, not localized swelling of the vulva.
Q.363. A 36-year-old woman complains of menorrhagia since 6 months associated with pelvic pain. All investigations and examination are normal. What is the treatment?
Correct Answer : B
In this scenario as all the investigations are normal, the most likely cause of mennorhagia is hormonal imbalance. COCPs contain both estrogen and progesterone, which helps regulate the mestrual cycle.
Q.364. What is best test for early pregnancy?
Correct Answer : A
USG is the most accurate and reliable test for detection of early pregnancy.
Q.365. A K/C/O DM wants to get pregnant. What is the management?
Correct Answer : A
Preconception care- Having good control of diabetes before conception is a crucial step for a healthy pregnancy. This involves managing blood sugar levels through diet, exercise, and medication if needed.
Q.366. Patient with malodourous greyish white vaginal discharge with clue cells seen on. What is the diagnosis?
Correct Answer : A
Clue cells are a typical finding in - Bacterial vaginosis
Q.367. What is most benign cause of postmenopausal bleeding?
Correct Answer : B
Atrophic vaginitis is a common condition caused by decreased estrogen levels after menopause.
Q.368. A patient complains of infertility since 6 years and dysmenorrhea. What is the likely diagnosis?
Correct Answer : A
The combination of infertility and dysmenorrhea strongly suggests endometriosis as the likely diagnosis.
Q.369. A married woman presented with hirsutism and obesity. X ray showed ovarian cyst. She wants to conceive. What is the best treatment?
Correct Answer : A
Clomiphene citrate is an ovulation inducing medication, it regulates ovultion and increases the chances of pregnancy.
Q.370. A woman presented with right sided abdominal pain with B-HCG 5000. LMP - 5 weeks ago. Patient is hemodynamically stable. What is the treatment?
Correct Answer : B
As the women is hemodynamically stable - Methotrexate is the drug of choice.
[Hemodynamically unstable - Surgery is the management of choice]
Q.371. Fetal distress occurs in which of the following?
Correct Answer : B
Late deceleration are a type of fetal heart rate pattern where the heart rate slows down after a contraction begins and doesn't return to normal until after the contraction ends. This pattern can indicate that the placenta isn't delivering enough oxygen to the baby, which is a sign of fetal distress.
Q.372. A patient 2 hours after delivery has severe vaginal bleeding. BP- 90/70mmHg. What's the initial management?
Correct Answer : B
IV fluids should be started first as the patient is not hemodynamically stable.
Q.373. A lactating mother complains of fever, breast tenderness and redness. What is the treatment?
Correct Answer : A
Diagnosis - Mastitis
- This is the best course of management in a lactating mother with mastitis.
Q.374. A female G3P0, presented with infertility, has regular non heavy cycles. She has a past history of Trichomonas infection treated at the age of 17, previous 3 elective D/C in first month of gestation. What is diagnosis?
Correct Answer : A
Ashermann's syndrome is characterized by formation of adhesion due to formation of scar tissue caused by previous uterine procedures like D and C as seen in this patient's case.
Q.375. A 40-year-old presented with heavy and intercycle bleeding. She is not pregnant, and not on hormonal contraceptives. What's the cause?
Correct Answer : A
The patient's age and nature of her bleeding makes an anovulatory cycle the most likely cause.
Q.376. Which of the following is the most concerning sign in pregnancy?
Correct Answer : A
Vaginal bleeding is the most concerning sign in pregnancy due to the potential of serious complications like miscarraige, ectopic pregnancy, and placental problems.
Other options are less concerning but need monitoring.
Q.377. A female presented with defemenization (breast atrophy and deepening of voice) and found to have ovarian cancer. What is the possible type?
Correct Answer : C
The defaminization symptoms strongly suggest that there is precense of androgens, Sertoli-Leydig cell tumors are known of producing androgens, making them the most likely option.
Incorrect options-
- Thecoma- produces excess estrogen.
- Granulosa cell tumor- produces excess estrogen production.
- Stromal cell tumor- depends upon type of stromal cells involved.
Q.378. Relative contraindication of ergotamine for the treatment of post-partum hemorrhage is?
Correct Answer : C
Ergotamine can raise blood pressure. Using it in a woman with high BP after childbirth coul further increase her BP and cause serious complication.
Q.379. Which of the following statement is true regarding dT in pregnancy?
Correct Answer : A
dT is a vaccine used for protection against tetanus, a serious bacterial infection. Pregnant woman are actually at an increased risk of tetanus, and getting vaccinated against it will cater protection for both mother and fetus.
Q.380. A 28-year-old diabetic female who is married wants to become pregnant. Her blood glucose is well controlled, but she's concerned about congenital anomalies in her baby if she conceives. When should she start DM control?
Correct Answer : A
Pre conception control is essential as having high blood sugar levels in the early stages of pregnancy can increase the risk of birth defects in the baby.
Q.381. A 43-year-old lady complaints about non itchy, white, odorless, minimal vaginal discharge after intercourse, she isn’t using any contraceptive or vaginal douche. What is the diagnosis?
Correct Answer : D
Vaginal douche after intercourse can be adviced.
Q.382. A female patient is on antiepileptic drug. She wants to become pregnant. What well you advice her regarding epilepsy?
Correct Answer : A
Some antiepileptic medication can cause fetal malformation. This risk depends on certain medication and dose.
Q.383. A pregnant female developed gestational diabetes that was not controlled by diet and was switched to insulin. What is she at great risk to develop later in life?
Correct Answer : B
GDM especially in which insulin is needed, these females are at a higher risk of developing type 2 diabetes later in life.
Q.384. What is the treatment of gonorrhea?
Correct Answer : A
It's a single dose injectable antibiotic that is higly effective against Neisseria gonorrhoeae.
Q.385. A 39-year-old female presented with left sided abdominal pain, no dysuria or changes in bowel habit. She has a previous history of hysterectomy 4 years back but ovaries and tubes were preserved. On examination: abdomen was tender but no guarding. Investigations show leukocytosis and few pus cells in urine. There was also history of unprotected coitus with multiple partners. What is the management?
Correct Answer : B
Gonorrhea is an STI caused by bacteria Neisseria gonorrhoeae. It is typically treated with a course of antibiotic.
Q.386. Female wants a temporary contraceptive method. Which one of the following methods would you recommend her?
Correct Answer : A
These are very common and effective form of birth control. They work by preventing the release of ovum from the ovary. OCPs are easily reversible and can be stopped at any time.
Q.387. A female comes to the clinic with positive urine pregnancy test at home, what investigation would you do next?
Correct Answer : A
This is the most accurate way to confirm pregnancy.Home pregnancy tests detect the precense of hCG in urine but the blood tests can measure the exact level of beta hCG in blood. This helps determine how far along the pregnancy is and can rule out potential complications such as ectopic or molar pregnancy.
Q.388. A 43-year-old female has irregular menses since 3 months & complains of spotting since 1-2 days. What to do next?
Correct Answer : C
If FSH levels are high - confirms menopause.
Q.389. Mother after delivery complains of having low mood, depression, crying a lot for last 1 week, but she is now fine. What is the diagnosis?
Correct Answer : A
Maternal blues - Transient mood swings, tearfullness, mild depression within the first week after delivery, resolves on its own in 1-2 weeks.
Incorrect options-
- Postpartum depression- persistent low mood, lack of energy, and difficulty bonding with baby, lasting longer than 2 weeks.
- Postpartum psychosis- severe condition acommpanied by hallucinations, delusions, or erratic behaviour; requires urgent medical attention.
- Anxiety- excessive worry or fear that may occur with depression but lacks the hallmark tearfulness and low mood of maternal blues.
Q.390. Which is the most benign cause of vaginal bleeding in menopausal women?
Correct Answer : D
Atrophic vaginitis is a condition that occurs when estrogen levels in body drop after menopause. This can cause vaginal tissues to become thinner and drier, which can lead to vaginal bleeding.
Q.391. What is the most concerning thing about placental abruption?
Correct Answer : B
Placental abruption is defined as premature sepration of placenta from the uterine wall. This sepration can block the flow of oxygen, and nutrients to the baby, leading to fetal distress.
Q.392. A 54-year-old female with chronic pelvic pain is found to have a right sided ovarian mass. After the initial evaluation, surgery is planned to remove the mass. To avoid excessive bleeding during the surgery, the surgeon should ligate which of the following structures?
Correct Answer : B
Suspensory ligament of ovary should be ligated as it contains -
- ovarian artery
- ovarian vein
- ovarian plexus and lymphatic vessels
Q.393. In which condition should we stop the Oral Contraceptive Pills?
Correct Answer : A
OCP side effects:
- Venous thromboembolism
- Increased risk of breast cancer (while decreasing the risk of ovarian, endometrial & colon cancers)
- Weight gain
- Acne
- Depression
- Hypertension
Q.394. An infertile female gives h/o unprotected sexual intercourse 3-4 times weekly. She presents with h/o pelvic pain. On examination, tender nodules on the uterosacral ligament are felt. What is the best investigation?
Correct Answer : A
It is the best investigation for this patient because it allows direct visualization of the organs and can help identify abnormalities, such as endometriosis or PID, which cause pelvic pain and infertility.
Q.395. A 42-year-old pregnant female, in her 2nd trimester, concerned about having a baby with Down syndrome. What's the non invasive test to rule out?
Correct Answer : A
Triple test is a non-invasive test that measures the level of 3 substances in a pregnant woman's blood- AFP, hCG, and estriol. These levels can indicate an increased risk of Down syndrome. The triple test is typically performed between 15-20 weeks of pregnancy.
Q.396. What does mother's milk have high proportion of when compared to cow's milk?
Correct Answer : D
Protein is a crucial component for building and repairing tissues, and mother's milk contains significantly more protein than cow's milk.
Q.397. Which of the following is true regarding pregnancy in a woman with thyroid disease?
Correct Answer : A
Anti-thyroid medications are used to manage thyroid disorders, as they can have several adverse effects leading to complications in the developing fetus as they have the tendency to cross the placenta. Anti-thyroid medications include- methimazole, propylthiouracil, and carbimazole.
Q.398. A woman with breech presentation + dilated cervix + wide pelvis + late deceleration. How will you manage?
Correct Answer : C
As the baby is in breech presentation and there is precense of late decelaration the most appropriate step to go further in this patient is to perform a C-section.
Incorrect options-
- Forceps delivery is not recommended for breech position.
- Spontaneous delivery is not advisable in breech presentation especially in precence of late decelaration which is indicative of fetal distress.
- Vaccum delivery is not suitable for breech preentation.
Q.399. Female patient uses douches excessively in the past 6 weeks, she developed a malodorous vaginal discharge, no itching, no urinary symptoms. On examination revealed pooling in vaginal wall, milky discharge with intact vulva. PH is 6. KOH revealed clue cells, no yeast, no hyphae. Which of the following is NOT accepted to give this patient?
Correct Answer : D
The symptoms given in the scenario is strongly suggestive of bacterial vaginosis.
Miconazole cream is an antifungal medication and will not be useful in treating bacterial infection like bacterial vaginosis.
Q.400. A female presented with vaginal discharge, and itching. On microscopic examination - mycelial cells and spores. This condition is most likely to be associated with which of the following?
Correct Answer : B
Vaginal thrush is a common infection caused by a yeast called Candida albicans. Vulvovaginal candidiasis is usually secondary to overgrowth of normal flora Candida species in the vagina. Conditions that interrupt the balance of normal vaginal flora include antibiotic use, oral contraceptives, contraceptive devices, high estrogen levels, and immunocompromised states such as diabetes mellitus and HIV. Women are prone to vaginal thrush between puberty and menopause because, under the influence of the hormone estrogen, the cells lining the vagina produce sugar and yeasts which Candida albicans are attracted to. That is why thrush is rare before puberty.